Está en la página 1de 954
Contents ==7 Functions and Models 1 1.1 Four Ways to Represent a Function 1.2 Mathematical Models 9 1.3. New Functions from Old Functions 14 1.4 Graphing Calculators and Computers 25 Review 34 Principles of Problem Solving 39 ==2 Limits and Rates of Change 43 21 The Tangent and Velocity Problems 43 22 TheLimit ofaFunction 45 2.3. Calculating Limits Using the Limit Laws $3 24 — The Precise Definition of a Limit 61 2.5 Continuity 68 2.6 Tangents, Velocities, and Other Rates of Change 76 Review 82 Problems Plus 89 = 3 Derivatives 93 1 Derivatives 93 3.2 ‘The Derivative as a Function 98 3.3. Differentiation Formulas 107 3.4 Rates of Change in the Natural and Social Sciences 118 3.5 Derivatives of Trigonometric Functions 126 3.6 The Chain Rule 130 3.7 Implicit Differentiation 137 3.8 Higher Derivatives 145 Applied Project © Where Should a Pilot Start Descent? 153 9 Related Rates 154 3.10 Linear Approximations and Differentials 161 Laboratory Project © Taylor Polynomials 167 Review 170 Problems Plus 181 woo CONTENTS ==4 Applications of Differentiation 191 41 42 43 44 48 4.6 47 48 49 4.10 Maximum and Minimum Values 191 Applied Project © The Calculus of Rainbows 201 The Mean Value Theorem 202 How Derivatives Affect the Shape of a Graph 206 Limits at Infinity; Horizontal Asymptotes 219 Summary of Curve Sketching 230 Graphing with Calculus and Calculators 249 Optimization Problems 263 Applied Project © The Shape of aCan 278 Applications to Economics. 279 Newton’s Method 284 Antiderivatives 292 Review 300 Problems Plus 319 =5 Integrals 325 Sa 8.2 33 34 3.5 Areas and Distances 325 ‘The Definite Integral 333 Discovery Project = Area Functions 342 The Fundamental Theorem of Calculus 344 Indefinite Integrals and the Total Change Theorem 353 ‘The Substitution Rule 358 Review 366 Problems Plus 375 = Applications of Integration 385 61 6.2 63 64 65 Areas between Curves 385 Volumes 397 Volumes by Cylindrical Shells 412 Work 421 Average Value ofa Function 424 Review 426 Problems Plus 433 Conrents GO vit =7 Inverse Functions: Exponential, Logarithmic, and Inverse Trigonometric Functions 441 7.1 Inverse Functions 441 457. ‘Deriv hes of Logatithmte 4 ions: 463 7.8 Inverse Trigonometric Functions 497 Applied Project © Where to Sit at the Movies 507 7.6 Hyperbolic Functions $08 7.7 Indeterminate Forms and L'Hospital’s Rule 515, Review 529 Problems Plus 543 Techniques of Integration 547 8.1 Integration by Parts 547 82 Trigonometric Integrals 587 8.3 Trigonometric Substitution 564 84 Integration of Rational Functions by Partial Fractions 573 8.5 Strategy for Integration $86 8.6 Integration Using Tables and Computer Algebra Systems 595 Discovery Project 3 Patterns in Integrals 603 8.7 Approximate Integration 606 88 Improper Integrals 619 Review 632 Problems Plus 647 ==9 Further Applications of Integration 653 9.1 ArcLength 653 9.2 Area of a Surface of Revolution 661 Discovery Project © Rotating on a Slant 669 9.3 Applications to Physics and Engineering 670 9.4 Applications to Economics and Biology 682 9.5 Probability 688 Review 687 Problems Plus 693 ee vill CONTENTS == Differential Equations 701 10.1 10.2 10.3 10.4 10.5 10.6 10.7 Modeling with Differential Equations 701 Direction Fields and Euler's Method 703 Separable Equations 710 Applied Project © Which Is Faster, Going Up or Coming Down? 719 Exponential Growth and Decay 720 Applied Project 3 Calculus and Baseball 724 ‘The Logistic Equation 726 Linear Equations 734 Predator-Prey Systems 739 Review 743 Problems Plus 751 SFI Parametric Equations and Polar Coordinates 757 dt 112 113 114 11s 116 17 Curves Defined by Parametric Equations 757 Laboratory Project 0 Families of Hypocycloids 767 Tangents and Areas 770 Laboratory Project 0 Bézier Curves 780 Are Length and Surface Area 781 Polar Coordinates 788 Areas and Lengths in Polar Coordinates 803 Conic Sections 813 Conic Sections in Polar Coordinates 820 Review 825 Problems Plus 835 =F Infinite Sequences and Series _841 124 12.2 12.3 12.4 12.5 12.6 127 12.8, Sequences 841 Laboratory Project © Logistic Sequences 850 Series 854 ‘The Integral Test and Estimates of Sums 865 ‘The Comparison Tests 871 Alternating Series 875 Absolute Convergence and the Ratio and Root Tests 879 Strategy for Testing Series 884 Power Series 887 12.9 12.10 12.11 12.12 contents ix Representations of Functions as Power Series 893 Taylor and Maclaurin Series 901 The Binomial Series 912 Applications of Taylor Polynomials 919 Applied Project © Radiation from the Stars. 931 Review 932 Problems Plus 943 amyor> Appendixes 951 Intervals, Inequalities, and Absolute Values 951 Coordinate Geometry and Lines 956 Graphs of Second-Degree Equations 962 Trigonometry 967 Sigma Notation 975 Complex Numbers 978 wa Functions and Models =! Four Ways to Represent a Function \n exercises requiring estimations or approximations, your answers may vary slightly from the answers given here. 1. (@) The point (—1, —2) is on the graph of f, so f (—1) = (©) Whenx =2, yis about 28, so f (2) 2.8. © SG) =2is equivalent to y = 2. When (@ Reasonable estimates for x when y = 0 arex = 2, wehavex = —3 and 3. © The domain of f consists ofall x-values onthe graph of /- For this function, the domain is -3 x > 6orx <0, 50 the domain is, 20,0) 16,00) 26, h(x) = YT=Br is defined when 7 ~ 3x 2 or x < 3,30 the domain is (~20, ] 21. f(t) = Ye is defined for every r, since every real number has a cube root. The domain is the set of all real faumbers, R. 28. h(x)=VE=e Nowy=VE=w = ade @ ey so the graph is the top half ofa citele of radius 2 with center atthe origin. The 2 domain is {x | 4— x? > 0} = [2,2] From the graph, the range is 0< y <2, or [0, 2]. a Bf 2x, Domain is B. ” 30. f (3) = 22 + 2x = 1 = (x? $20 41) —2 = (a + 1)? 2, sothe graphs » “put itera (-.-3) Te done y a oa 31. g (x) = YEH is defined when x ~$ & Oorx = 5, so the domain is , {5, 00). Since Je-5 => Pux-5 = xrays, ‘we see that g is the top half of parabola. SECTION 1.41 FOURWAYS TOREPRESENTA FUNCTION C5: 32. (2) = VO—2x. The domain is (x | 6 — 2x > 0) = (-c0, 3). 7 33. G (x) = [x] +x. Since [x we have x ifx>0 x ifx <0 oe x+x ifx>0 [2 ifx>o ac 0 ifx <0 7 Domain is R, Note that the negative x-axis is part of the graph of G. mxtx ifx<0 7 de if2e20) [2 ite 20 2x if2x<0| | -2 ifx <0 | Domain isR ap . 20] _[ titeoo Bsw=t= - POAT] [aco veo {2 ite <0 Note that we did not use x > 0, because x #0. Hence, the domain of f is (1x £0) +342 x42 +2 Domain is (x | x =2). The hole in the graph can be found using the simplified function, f(x) = x +3. h (=2) = | indicates that the hole has coordinates (-2, 1). 80 for x fax $3. x itr0 7 3B-x ifxe-1 Domain is R. Domain is B. 6 © GHAPTER1 FUNCTIONS AND MODELS - ifx< W.fG)=) 3x42 if-t 1, the graph is the line with slope 1 and x-intercept 1, that is, the line 2 itx<0 y=1(x=1)=x~1. Sothe finctionis f(x) =| —2e+2 if0 0, since lengths must be postive quatitis. SECTION 141 FOURWAYSTOREPRESENT AFUNCTION © 7 Let the length of a side of the equilateral triangle be x. Then by the Pythagorean Theorem, the height y of the 2 gle satisfies y? + (Jx) = 27, sothat y = fx, Using the formula for the area A ofa triangle, A=} (base) (height), we obtain A (x) = (4) (8x) = 4x2, with domain x > 0. Let the volume ofthe cube be V and the length of an edge be L. Then " = L? so L = 3/7, and the surface area is 2 5) =6( 7) = 6722, with domain 7 > 0 5. Let each side of the base of the box have length x, and let the height ofthe box be h. Since the volume is 2, we know that 2 = hx, so that h = 2/x?, and the surface arca is S = x? + 4x. Thus, S(x) =x? 44 (2/22) = 3? + 8/x, with domain x > 0 2 xe 52. The area of the window is A = xh + Lar () = xh + >, where isthe height ofthe rectangular portion of the windows The perimeter is P =2h x4 4ax=30 © 2h=a30—x—Jax 4 he} (60-2e—a3). xx FE asx - fx? - te & Since the lengths x and t must be positive quant |, wehavex > Oandh > 0. Fork > 0, wehave 2h > 0 30-x-Jax>0 @ W>2e+nx re Hence, the domain of 4 is 0 <> 20-250 x<10;w>0 e& 12-2x>0 © x <6;andx > 0. Combining these restrictions gives us the domain 0 * 2@ (©) The slope of $ means that F inereases § degrees for eaeh increase of 1°C. (Equivalently, F increases by 9 when C reases by 5 and F decreases by 9 when C' decreases by 5.) ‘The Fintercept of 32 isthe Fahrenheit temperature ‘corresponding to a Celsius temperature of 0. 8. (a) Let d = distance traveled (in miles) and ¢ = time elapsed (in hours), Att =0,d =O and ats = SOmimuies = 50: z= $h, 41 = 40, Ths we have vo points: (0,0) and (§, 40), s0 m= o> = Bund sod = AB (©) The slope is 48 and represents the car's speed in mi/h. SECTION Ta NuTHEWATCALWODRS T= 15 © IN-B ee Pede = 51.13) 9. (@) Using 1 in place of x and 7 in place of y, we find the slope to be Soa linear equation is? -80=1(N -173) > T-80 (6) The slope of mean tha the temperature in Fabrenbet degrees increases one-sixth as rapidly asthe number of ticket chins per mute Said diferent, each inerease of 6 cricket chirps per minute coresponds to an increase of 1°F, (© When V = 150, the temperature is given approximately by 7 = 1 (150) +2 = 16.16°F ~ 76°F. 10. (a) Let x denote the number of chairs produced in one day and y the associated cost. Using the points (100, 2200) and (300, 4800) we get the slope $9=72 = 360 = 13. Soy —2200 = 13(x - 100) <> y= 13x +900, () The slope ofthe line in pat (a) is 13 and it represents the cost of producing each additional chit (©) The y-intercept is 900 and it represents the fixed daily costs of operating the factory change in pressure _ 4.34 10 feet change in depth ~ “10 (4, P) = ©, 15), wehave P15 =0.434 (4-0) > P =0.434d 415, (b) When P = 100, then 100 = O.434d-+15 <> O434d=85 o> d= 195.85 feet. Thus, the pressure is 100 1b/in? ata depth of approximately 196 feet. 11. (@) Weare given = 0.434. Using P for pressure and d for depth with the point a 460-380 80 ed ~ BO0—480 ~ 320 4a +260. (b) Letting d = 1500 we get C = } (1500) + 260 = 635. The cost of driving 1500 miles is $635. 12, (@) Using d in place of x and C in place of p, we find the slope to be linear equation is C ~ 460 =} (d- 800) «2 C-460=4d-200 © C © "t ‘The slope ofthe line represents the cost per mile, $0.25 ba (@) The intercept represents the fixed cost, $260. } (©) Because you have fixed monthly costs such as insurance 00 ‘and car payments, a8 well as costs thet inerease as you fe lene mite : for each additional mile driven is a constant, toe 18, (a) The data appear to be periodic and a sine or cosine function would make the best model. A model of the form. J (x) = ae0s (bx) + € seems appropriate, (b) The data appear to be decreasing in a linear fashion. A model ofthe form f (x) = mx +b seems appropriate. 14. (a) The data appear to be increasing exponentially. A model of the form f (x) = appropriate, (b) The deta appear to be decreasing similarly to the values of the reciprocal funetion, A model of the form F (x) = a/x seems appropriate, +BY or f (2) = a+" +c seems 12 © GHAPTERT FUNCTIONS AND MODELS ‘Some values are given to many decimal places. Thase re the results given by several computer algebra systems —rounding is left tothe reader 5 @) (©) Using the points (4000, 14.1) and (60,000, 8.2), we obtain 82-141 i Y= 141 = Be Gop agp & ~ 1000 oF equivalently, y% -0,000105357x + 14.521429. 61000 ‘A linear model does seem appropri. a 6.000 (©) Using # computing device, we obtain the least squares regression line y = —0.0000997855x + 13.950764. (@) When x = 25,000, y * 11.456; oF about 11.5 per 100 population, (€) When x = 80,000, y ~ 5.968; or about a 6% chance. (@ When x = 200,000, y is negative, so the model does not apply. se +230 (chirps/min) o 270 (chinpsmin) \ s 45 os cP) ‘Using a computng deve, we obtain te least squares regression line y = 4.856r — 220.96. (c) When x = 100°F, y = 264.7 = 265 chirps/min. 7.) a9 ® me in KJ 2000 ext " 2000 em) ‘A linear model does seem appropriate. ‘Using 2 computing device, we obtain the least squares regression line y = —158.2403249x + 0.089119747, where «is the year and y is the height in feet. (©) When x = 2000, y = 20.00 8 (@® When x = 2100, y ~ 28.91 ft, This would be an increase of 9.49 ft from 1996 to 2100. Even though there was an increase of 8.59 ft from 1900 to 1996, itis unlikely that a similar increase will occur over the next 100 years SECTION1.2 MATHEMATICAL MODS 13 18, By looking atthe scatter plot of the data, we rule out the linear and logarithmic models. 9 Coin ednion ®) Seater plot We try various models: Quadratic; y = 0.496x? — 62.2855x + 1970.63 Cubic: y= 0.0201243201x* — 3.88037296x? + 247.6754468x — 5163.935198 Quart ¥ = 0.000295 1049x4 ~ 0,0654560995x> + 5.27525641x? — 180.2266511x + 2203.210956 Exponential: y = 241422994 (1,084516914)" Power: y= 0.00002285497]x3-16078251 610 cmon 619 conn) 69 Coan) * “0 of 19 ws Ko edacion (Reousion ) escton 6) ‘Quadeatc model Cubic model Quanic model 10 comin 69 Coe) After examining the graphs of these ‘models, we see that the cubie and quartic models are clearly the best. #¢ 0 , << no eceion*) adcion) ‘Exponential mode! Power model ° 19, 6000 gations) us 1g 8 computing device, we obtain the cubie funetion y= ax? + bx? ex +d with a = 0.00232567051876, =13.064877957628, ¢ = 24,463.10846422, and dd = ~15,265,793,872507. When x = 1925, y = 1922 (oil 1804 2010 yew 14S CHAPTER FUNCTIONS AND MODELS 20. (a) 20) eer) [| u “0 10 ‘The graph of T vs. d appears to be that of @ power function and the graph of In vs. Ind appears to be linear, 80 a power model seems reasonable. (b) T = 1,000396048a"-#%661718 (©) The power model in part (b) is approximately T = dS. Squaring both sides gives us 7? = matehes Keplet’s Third Law, 7? = ka , $0 the model 53 New Functions from Old Functions 4. (a) Ifthe graph of / is shifted 3 units upward, its equation becomes y = f (x) +3. (b) If che graph of / is shifted 3 units downward, its equation becomes » = f (x) ~ 3. (©) the graph of / is shifted 3 units to the righ, its equation becomes y = f (x ~ 3). (@) Ifthe graph of F is shifted 3 units to the lef, its equation becomes y= f ( +3). (©) Ifthe graph of / is reflected about the x-axis its equation becomes y = —f (x) ( Ifthe graph of / is reflected about the y-axis, its equation becomes y = f (x), (2) Ifthe graph of / is stretched vertically by a factor of 3, its equation becomes » = 3/ (x). 47 @). 2. (a) To obtain the graph of » = $f (x) ftom the graph of y = f(x), stretch the graph vertically by a factor of 5. (b) To obtain the graph of y = f (x ~S) from the graph of y = (x), shift the graph $ units to the right (©) To obiain the graph of y = —f (x) from the graph of y = f (x), reflect the graph about the x-axis (@) To obtain the graph of y = ~$/(x) from the graph of y = (x), sietch the graph vertically by a factor of S and reflect it about the x-axis. (6) To obiain the graph of y = / (Sx) fom the graph of y = /(x), shrink the graph horizontally by a factor of S. (9) To obtain the graph of y = 5,/(x) —3 from the graph of y =f (x), stretch the graph vertically by a factor of $ and shift it 3 units downward. (h) Ifthe graph of / is shrunk vertically by «factor of3, its equation becomes y 3. (a) (graph 3) The graph of / is shifted 4 units to the right and has equation y = f (x ~ 4). () (graph 1) The groph of / is shifted 3 units upward and bas equation y = J (x) +3. (©) (araph 4) The graph of / is shrunk vertically by a factor of 3 and has equation y = 4 F (x). @ (raph 5) The graph of / is shifted 4 units to the left and reflected about the x-axis. Its equation is, ya -fG +4). () (oraph 2) The graph of fis shifted 6 units to the eft and stretched vertically by a factor of 2 Its equation is y=2s@ +6) 4 (@) To graph y = f(x +4) we shift the graph of f 4 units to the left (©) To graph y = 2/ (x) we stretch the graph of f vertically by a factor of 2, pa] 5, (@) To graph y = f (2x) we shrink the graph of J horizontally by a factor of 2 (© To groph y = f(x) we reflect the graph of / about the y-axis, SECTION L3 NEWFUNCTIONS FROM OLO FUNCTIONS 15 (b) To graph y = f(x) +4 we shift the graph of f 4 units upward (@) To graph y = 1 (x) +3, we shrink the graph of f vertically by a factor of2, then refet the resulting graph about thesis shen shift the pad resulting graph 3 units u ry z LAS (b) Tognphy = /(Js) wesc the graph off horizontally by a factor of 2 F(x) we reflect the graph of (@) To graph y about the y-axis, then about the x-axis 6. The graph of y = (x) = V3x =x has been shifted 2 units to the right and stretched vertically by a factor of 2. ‘Thus, a function describing the praph is ya2se-2 f=) 9 23-6 (2 4x 4d) = 2VaaP Te 10 16 © CHAPTER FUNCTIONS AND MODELS 7. The graph of y = f (x) = V3x — x7 has been shifted 4 units to the left, reflected about the x-axis, and shifted ddowaward { unit. Thus, @funetion deseribing the graph is yack set) =1 Se reflect shift shift about 4 units unit seaxis left down ‘This function can be written as -V3@ 44) - $4 =I -fG44)- =a (PIG) ~ 1 = Va 8. (a) The graph of y = 2sinx can be obtained from the graph of (b) The graph of y + ¥¥ canbe p= sin.x by stretching it vertically by a factor of 2 obtained from the graph of y = V/¥ by shifting upward 1 un i a 7 10, y = 2 — cos: Start with the graph of y = cos.x, reflect about the x-axis, and then shift 2 units upward, a = ‘= tn 2x: Start with the graph of y = tan.x and compress horizontally by a factor of 2. youn de SECTION 1.3. NEWFUNCTIONS FROM OLD FUNCTIONS 17 12. y = YEFE: Start with the graph of y = YF and shift 2 units tothe left. Weyer t2xt3 shift 2 units upward, 92 4 De 41) 42= (+E +2: Start with he graph of y = 22, shift unit left and then 16. y = 2 sinzx: Start with the graph of y = sinx, compress horizontally by a factor of z, stretch vertically by a factor of 2, and then reflect about the x-axis yr sine ——_——— 18 © CHAPTER FUNCTIONS AND MODELS $ sin (e — &): Start withthe graph of factor of 3. sin.x, shift § units to the right, and then compress vertically by a >t y=sinlx 2) ya tain(e 198. ya 14 2e xP a? pO 1 a (eP—2e HI LE shift 1 nit right, reflet about the x-axis, and then shift 2 units upward == ” "Lys -te- OT % y= VFA — 3: Start with the eraph of and then shift 3 units downward 2— VET: Start with the graph of y = J, reflet about the x-axis, sift 1 unit to the left, and then shift 2 units upward, SECTION 3 NEWFUNCTIONS FROM OLD FUNCTIONS O19 y= (— 1) +2: Start with the graph of » shift 1 unit to the right, and then shift 2 units upward. 23, y = Ils] ~ 1: Start with the graph of y = [x, shift 1 unit downward, and then reflect the part of the graph from x =I tox = 1 about the x-axis, 24, y = Joos x|: Start with the raph of y = coss and reflect the parts ofthe graph that lie below the x-axis about the 25, This is just like the solution to Example 4 except the amplitude of the eurve is 14 ~ 12 = 2, So the function is LW = 1242: [35 ( ~80)}. March 31 isthe 90th day ofthe yar, so the model gives J (90) © 12.34 h. The daylight time (5:51 A.M. to 6:18 P.M.) is 12 hours and 27 minutes, or 12.45 h. The model value differs from the actual value by 1246-1234 0,00, less than 1%, 26. Using a sine function to model the brightness of Delta Cephei asa function of time, we take its period to be 5.4 days its amplitude to be 0.35 (on the scale of magnitude), and its average magnitude to be 4.0. If we take ¢ = ata time of average brightness, then the magnitude (brightness) as a funtion of time ¢ in days ean be modeled by the formula I (1) = 4.04 0.38sin (342). 21. (a) To obtain y = f ({x1), the portion of J (x) tothe right ofthe y-axis is reflected about the y-axis. (b) y = sin] © y 200 CHAPTER FUNCTIONS AND MODELS 28, The most important features of the given graph are the x-intercept andthe ‘maximum and minimum points. The graph of y= 1 / f(x) has vertical asymptote a the-x-values where there are x-intercepts on the graph of y = f (x). The maximum of I on the graph of y =f (x) corresponds to a minimum of 1/1 =1ony =1/f («). Similarly, the minimum on the graph of y= f(x) corresponds to a maximum on the graph of y = 1 /f (x). 28. Assuming that successive horizontal and vertical grilines are a unit apart, we can make a table of approximate ‘values as follows, x oy! J2 |3 |4 15 16 fe) 2/17} 13|10]07]03| 0 2) 2}a7|3 j2s}a4] 17 P@ tee) | 4] 44) 43/38] 34) 20 Connecting the points (x, /(x) + 2 (x) with a smooth curve gives an approximation to the graph of f + g. Extra points can be plotted between those listed above if necessary. 30, First note that the domain of f + g isthe intersection of the domains of f and g: that is, 7 +g is only defined where both f and g are defined. Taking the horizontal and vertical units of length to be the distances between successive vertical and horizontal gridlines, we can make a table of approximate values as follows: x —2[-1 [0] 1 [2 ]25]3 Te) -1) 22| 20] 24] 27] 27) 23 20) 1| -13| -12| -06 | 03 | 05 | 07 Fete} of 09| 08} 18] 30] 32] 30 Extra values of x (like the value 25 in the table above) ean be added as needed. 3. f(s) <9 +282 ge) = 38? — LD =R for both f and g (f+) (2) = 4208 43x Lax pS? 1, DER, (9) @) =? + 2x? — Gx? 1) x8 — 37 41, DER, (fe) (2) = (0 +224) (x? = 1) = 3x5 4 6x4 —F 27, D=R. 3420 (Qo a 3a [x |x #45} since 3x? - 1 #0, SECTION 3 NeWFUNCTIONS FIOM OLOFUNCTONS O21 @ f:) = VTFE,D=[-+1,20)3 8) = THs, D= (0, 1) (+ 8)@) = VIF8+ VI=x, D = 00, NN -1,00) = (1.11 ((-a)) = VFS VIB Dah) (eo) @) = JTFS. Jims = vT= x, D=[-1, 1} L) oy TEE £ De=[-1,1), Wemustexclidex = {since it would make £ undefir (4) 9 = FEE, 0= 11,0. Wemustecdex = sine it wou mate £ undefined BsWenewn ils Msw=se@) 35. f (x) = 2x4 — x; g(x) = 3x +2. D=R for both f and g, and hence for their composites. F (GG) = f Gx +2) = 23x +2)? ~ Bx +2) = 18x? + 2x +6. (f @)) = g Qe? 2) =3 2? =x) $2 = 6? — 3 +2 LLG) = F (2x? =x) = 2 (2x? ~ x)? = (2x? — x) = 8x4 BF 4x, GW) =£Gx42—3Gr+]+2— +E. 3. (6) = VE=T,D (fogs) =F ON) = Se") D=( ER | g(8) €[1,00)] = 00,1] U1, 00). Go) =e O)=2(v3 alee (Fo N@)=S/FO)= D={se([1,00)| VF=12 | (og) @) =e(e@) =e (x4) = (2) =x4,D=R. 3. f@)=1/x, D= lx |x 40} gs) = +2, D=R. (Low) = FO) = f(s? + 2x) = 1/ (2? +24), D= fr |x + 2x £0} = te |x FO} ee Ns) = BUS @) = 8 (W)x) = x? 42/x, D = te Lx #0). (fe) = SUG) = fs) = Fe =H D= le Lx £0) (eee) (x) = Bw) = g(x? + 2x) = (x9 +2)? $2 (3 + 2x) = 09 + x? +1205 + 1003 + de, DER, 1,00); g() =, =VF=T, x-1,D= {1,00} 1G) x-l tele # theG)= 5, Dab ls 4-1 (foa) i)-(i-4) Strix 4-1), ani Wet eo Ne= “VWenne1 (reNW) = fe 1x #12) @ox)(x)= D/@+0), = bebe 40-1 D/@+D+ 22 O_GHAPTERS FUNCTIONS AND MODELS 38. f) =sins, D=R; g@)=1- VF, (Fog) @) =f EG) = f (1 — v3) = sin (I~ v%), D = (0, 20). @o NG) = @) =gsinx) = | — vsinx. For VSinx to be defined, we must have sinx >0 9 x € [0,x], [2x, 3m], (-22,—m], (4x, 5x], [—4x, -32], -.., 30 D = (x |x © [Dar, x +2nr], where n isan integer) (Fo) = FF) = Fins) = sin(sinx), D=R (og) (x) =e(g (0) <8 (1 - va) =1- VI= Ve, D = {x 2011 - VF 2 0} = 10,1]. 4. f @) = VHT, D = (00, -11U1,00): 20) = D= (20,11 (Fog) &) = Sg) =f (VTHa) = (T= a) — | = VR To find the domain of (fo g) (x), we must find the values of x that are inthe domain of g such that g(x) is inthe domain of /. In symbols, we have D= {x € (co, I]| YT=¥ € (-00, 1] U[1, 00). First, we concentrate onthe requirement that VI=¥ € (00, -I]U LI, 00). Because YT== 2 0, T= is not in (00, —1]. If VT => is in [1, 00), then we musthave YT—¥ > 1 =} 1—x21 = x <0. Combining the restrictions x < 0 and x € (co, I], we obiain D = (00, 0} GoM) = @) =2(VP=1) = VI, da fre 00, -UU.09 1 VF=Te Co.) Now VFETSL = A a1s1 = 32 9 bls V2 = -VF 1=x¢1 = x20. Combining this restriction with x ¢ (~00, 1], we obtain D = [0,1]. AL (fogoh)(x) = Seb @) =Se@-I)=s(VE=1 = vE-T= 1 &@ (Sogo) = SEED) = Fe? +2)) =F ((e? +2)?) =1 [2 42? B.(fogoW x) =f GHEY) =F (@ (V5) =S (VE -3) = (VE-5)' +1 = (0, 00) M Foaoiney=semo=se(e=s( Let g(x) =x —9 and f (x) = 25, Then (f 22) @) = - 98 = FG). $6, Let g(x) = VF and f (x) = sinx. Then (f 0g) (x) = sin VX = F (2) an @) = og) = sy = FH) A. Let g(x) = x? and f (x) = =. Then (fog) (1) = STG =F () 4, Letg (x) = 3-43 and f (x) = 1/x. Then (f0g) (2) = 1/(x +3) =G@). SECTION 13 NEWFUNCTIONSFROMOLOFUNCTONS 23 49. Let g (0) = cose and f (0) = Vi. Then (fog) (1) = Veosi = u(0). 50. Let g(¢) = xt and f(t) = tant. Then (fog) (t) =tanzt =u (0) BH. Let h(x) =a, g(x) =3%, and f (x) = 1 =x. Then (fog oh)(x) =1-3" = A(x), 52 Let h(x) = VE, g(x) =x — Land f (x) = YX. Then (fog oh) (x) = Va —1 = He). 5B. Let h(x) = /%, g(x) = seex, and f (x) = x4. Then (fog oh) (x) = (sec /z)* = sec! (Vz) = HG). 54) FE (I) = FO) =5 ) 2) =e@) =2 OSE M =f B= @s@iy=s6 © ee NG =e(/G)) =2M)=1 © (Fon) =f) =/G)=4 5B. (a) g (2) = 5, because the point (2, 5) ison the graph of g. Thus, f(g 2)) = f (5) = 4, because the point (5,4) is, fon the graph of f. ®) eFO)=8@=3 © Fo = f(g) =/G)=0 @ Ge NO =£(F ©) =e ©). This value is not defined, because there is no point on the graph of g that has -x-coordinate 6, © @og)(-2) =e (2) OFONH=SF@) 56, To find a particular value of f(g (+)), say for x = 0, we note from the graph that g (0) ~2.8 and f (2.8) & —0.5 Thus, f (g ()) ¥ f 2.8) © —0.5. The other values listed inthe table were obtained in a similar fashion. x [eo | fey x[e@ | see) ; =s5|-02|-4 o| 28] -05 —4} 12| -33 t) 22] -17 : -3] 22) -17 2) 12] -33 * -2] 28] -05 3| -02 | -4 -1] 3 | -02 4| -19 | -22 s| nai] 19 57. (@) Using the relationship distance = rate tine with the radius ras the distance, we have r (2) = 601. (b) A= ar? => (Aor) (1) = 4 W)) =z (601)? = 3600207, This formula gives us the extent of the rippled area (in cm?) at any time ¢ BB G@)dar > d=350 (b) There is a Pythagorean relationship involving the legs with lengths d and 1 and the hypotenuse with length s: #42 22 Thus s(d) = VE FI. © (od) =5d()) =5 B502) = VGS0 FT 0 itr <0 58. (2) vo sal ® " 120 ift>0 : 120 30 ¥ () = 120H () 7 ; 24 GGHAPTER FUNCTIONS AND MODELS © v Starting with he formutain part (b), we replace 120 with 240 to reflet the aol = iret voage, A, borase wee sig Sunk the ight of | = 0, we replace with ¢~ 5. Thus, the formula is ¥ () = 24011 @ ~ 5). morome (fH a) R() =H » 1 itr20 veoefo He 7 = OF O=| y itocr< wo} so (= 2H (,t 5 60. oO voe{® 2? y OF O=) goon itrses 3 | 10 VW =A0-N HUD S32 tt 61. (@) By examining the veriable terms in g and h, we deduce that we must square g to get the terms 4x? and 4x in i. Ifwe let f(x) =x? +o, then (fog) ) = FEO) = SOx +1) = Ort IP tema? tax tte) Since h(x) = 4x2 + 4x 47, we must have 1-+¢ = 7. Soc = Gand g(x) =37 +6. (b) We need a function ¢ s0 that SGC) = 3 )) +5 = Ga) = 3x? 4323 (PH) $2 = 3 (07 +x ~ 1) 45. Some see that gi) ter] 62. We nced a function g so that ¢ (/ (2) = g (x +4) = h(x) = 4x — function g must be g (x) = 4x ~ 17. 4G: +4) — 17. So we see that the 63, We need to examine h (—x). hen) =o = SEER) =F (la)) [because giseven] = A(x) Because h (—x) = h (x), A is an even function, 64. h(=x) =f (@(-¥) = F (He (8). At this point, we can’t simplify the expression, so we might try to find a counterexample to show tha Ais not an odd function, Let g (x) = x, an odd function, and f (x) = x? + x. Then h(x) = x? +, which is neither even nor od. Now suppose / is an odd function. Then f (—g (x) = —f (g (2)) = ~h (2). Hence, fe (—x) = —Ha(x), and so is odd if both f and g are odd Now suppose / is an even function, Then f(—g (2)) =f (g (=)) = h(). Hence, (x) = F(x), and so is even if g is odd and / is even SECTION 14 GRAPHING CALCULATORS AND COMPUTERS O25 65. (2) P= (a, g(a) and Q = (& (a), g (a) beeause Q has the @ a) same y-value as P and itis onthe line y = x ' pos () The s-value of Q is g (a); this is also the xevalue of R. The _yevalue of Ris therefore f (x-value), thats, f(g (@)). Hence, R= (@ (a), f (@ (a) (©) The coordinates of $ are (a, f(g (a))) oF, equivalently, @h@). 66. We only need to plot points for the first quadrant since we ‘can see that / is an odd function, and by Exercise 64, we yoann then know that fo / is an odd funetion, and hence, symmetric with respect to the origin. x Jojos|1]is go jo} fas}isfo re |ofis| 14] rs fo EE) Graphing Calculators and Computers 1. fG)=xt42 () [2,2] by [-2,2] () [0.4] by [0.4] (4,4) by 4,4] -| 2 -f- 4 ‘ = o ‘ + @ — [8,8] by (4,40) (©) [-40, 40} by [-80, 800] — = Be ~The most appropriate raph is ‘produced in viewing rectangle (4), | 7” ‘ - 0 2 9. CHAPTER FUNCTIONS AND MODELS LsGaP +46 @ [-5,5] by [-5, 5] Tr oz (© [-15,8] by [-20, 100] 10 ‘The most appropriate graph is produced in viewing rectangle (c). Bf (e) = 10425x = 39 @ [-4, 4] by [-4, 4] 4 (©) [-20,20} by {—100, 100] 100 () © @ {0, 10) by [~20, 100} 100, 20 [-10, 3} by [-100, 20] 20 [-10, 10} by [—10, 10] 0 =o [-100, 100] by {-200, 200) 200, 7 ie ‘The most appropriate graph is produced in viewing rectangle (c) because the maximum and minimum points are fairly easy to see and estimate. SECTION 14 GRAPHING CALCULATORS AND COMPUTERS © 27 4 f@)=VErR @ [4.4] by [-4, 4] ©) [5,5] by 0, 100] , 100 ‘ -s s ° © [-10, 10} by [-10, 40) © L210) by 2,61 — go 10 0 a © a = ‘The most appropriate graph is produced in viewing rectangle (8). 5. Since the graph of f (x) = 5 + 20x — x? isa parabola opening 6 400 downward, an appropriate viewing rectangle should include the ‘maximum point. 10 6 5 -0|—] x” 0 7. f (x) = Y256—%* To find an appropriate viewing rectangle, we calculate /°s domain and range: 256x720 #4 x? < 256 e+ Ie] = 16 © 16 100. whenever x > 101. 300,00 af ooo ra LY] 200 “oa ft 10 31.(2) @ [0,5] by 10,20] Gi) {0,25} by (0,107) Git) (0, SO} by [0, 10°] s Asx gets large, f(x) = 2* grows more rapidly than 2 (x) = (b) From the graphs in part (2, it appears thatthe two solutions are x = 1.2 and 22.4, 32. @) @ [-4,4] by (0, 20] (ii) [0, 10} by [0, 5000) Gi) [0,20] by [0, 10°] 20 5000 107 + + ‘| oY 0 Asx gets large, f (x) = 3¥ grows more rapidly than g (x) (b) From the graphs in (0, it appears that the three solutions are x * ~0,80, 1.52 and 7.17. a os ‘We see from the graphs of y = |sinx —x| and y = 0.1 that there are two solutions tothe equation [sin — x] = 0.1: x © —0.85 and x ~ 0.85. The condition [sin x = x] < 0.1 holds for any x lying between these two values, 320 CHAPTER FUNCTIONS AND MODELS Px) = 3x5 — 523 42x, a 2 op 10000 Q(x) = 385. These graphs are 2 caer to Significantly different only inthe | region close to the origin. The larger a viewing reetangle one chooses, the ro00 ‘more similar the two graphs look. 35. (a) The root functions y= J, _(b) The root functions y = x, (© The root functions y = /%, yo Yea = yo Rudy= y= Wy = emdy= (4) For any n, the nth root of 0 is O and the mth root of 1 is 15 that i, all nth root funetions pass through the points (0,0) and (1,1). + For odd nthe domain ofthe nth root function is R, while for even n, it is (r € R | x 2 0} «Graphs of even root functions look similar to that of V/, while those of odd root fu Us. ‘+ Asm increases, the graph of 7% becomes steeper near 0 and latter for = > 1 Vx, y= 1), yeys yotjst 1/3 and y = 1/xt 36. (2) The functions y= 1/x and (b) The functions y = 1/x and (c) The functions |/x” pass through the point (1, 1). ly above the x-axis, The graphs of 1/x" for m even are similar (@) « ‘The graphs of al functions of the form If miseven, the graph of the function is ent to one another. ‘#1 mis od, the function is positive for positive x and negative for negative x. The graphs of 1/x” form odd are similar to one another. ‘¢ Asm increases, the graphs of 1/+” approach 0 faster as x 00, SECTION 14 GRAPHING CALCULATORS AND COMPUTERS 3 30. f (x) =x" + ex? +x. Ife < 0, there are three humps: two minimum points and a maximum point, These humps get flatter us c increases, until at ¢ = 0 two of the humps disappear and there is only one ‘minimum point. This single hump then moves tothe right and approaches the origin asc inerease. 3B. f(x) = TF ex". Ife < 0, the function is only defined on [-1//=e .1 / VE } and its graph is the top half of an ellipse. If =, the graph is the line y = 1. Ife > 0, the graph isthe top half ‘of a hyperbola. As ¢ approaches 0, these curves hecome fatter and approach the line y = 1 39, y= x"'2-*. As m increases, the 4s ‘maximum of the function moves further from the origin, and gets larger. Note, however, that regardless of n, the function approaches 0 as x > 00. ‘The “bullet” becomes broader as ¢ increases. 4. y? =ex3 +2? Ie <0, he loop isto the right of the origin, and if cs positive itis tothe let In both cass, the closer cis to 0, the large the oop is. (In the limiting case, ¢ = 0, the lop i “infinite”, that it doesnt close) Also, the larger |c| is, the steeper the slope is on the loopless side of the origin, #2 (a) y =sin (V8) wy This unetion is not period; it oseilates tess frequently asx increases in(x?) This function oscillates more frequently as |x} increases. Note also that this function is even, whereas sin x is odd, 34 © CHAPTER FUNCTIONS ANO MODELS =I Review CONCEPT CHECK ———————<— 1. (@) A function f i a rule that assigns to each element x in a set A exactly one element, called f (x), in a set B. ‘The set 4 is called the domain of the function. The range of / is the set of all possible values of f(x) as x varies throughout the domain (b) If / isa function with domain A, then its graph is the set of ordered pairs (x, f (x)) [x € A) (©) Use the Vertical Line Test on page 17. 2. The four ways to represent a function ar: verbally, numerically, visually, and algebraically. An example of each is given below. ‘Verbally: An assignment of students to chairs ina classroom (a description in words) ‘Numerically: A tax table that assigns an amount of tax to an income (a table of values) Visually: A graphical history ofthe Dow Jones average (a graph) Algebraically: A relationship between distance, rate, and time: d = rt (an expliet formula) 3. (a) Aneven function f satisfies f (—x) = f (x) for every number x in its domain. It is symmetric with respect 10 the yas. (©) An odd function g satisfies ¢ (x) the origin. ~# (x) for every number x in its domain, It is symmetric with respect 10 4 A function J is called increasing on an interval I if f (x1) < f (a2) whenever x1 < x2 in I. 5 A mathematical mode is a mathematical deserition (often by means of a function or an equation) of a real-world phenomenon, 6 (@ Linear function: f(x) = 2s +1.) (b) Power function: f(x) =x, f (x) 1 ane fisynx (c) Exponential function: f(x) = 2", f (x) =a" (4) Quadratic function: f (x) =x? +x +1, S (x) =ax? ox $e (€) Polynomial of degree 5: f(x) = x5 +2 ; -pey= a, pay 2 (f) Rational function: f Gx) = 5. Fe) ae) yhere P (x) and Q (x) are polynomials ae . ® y © » ym sing: yo" CHAPTERT HVIW a5 @ » © 1 © * in yo : y : '% (@) The domain of / + g is the intersection of the domain of f and the domain of g; that is, A B. (©) The domain of fg is also 40 B. (© The domain of //g must exclude values of x that make g equal to 0; that is, (x € AM Bg) #0). 10. Given two functions / and g, the composite function fog is defined by (/ © 2) (x) = f(g (x)). The domain of ‘F 08 isthe sot of all x in the domain of ¢ such that g (2) isin the domain of f. +2 S@)-2 (© Ithe graph of fis shifted 2 units to the right, its equation becomes » = f (x ~2) (@ Ifthe graph of fis shifted 2 units tothe lef, its equation becomes y = f (x +2). (© Ifthe graph of f is reflected about the x-axis, its equation becomes y = —f (x). ( Ifthe graph of f is reflected about the y-axis its equation becomes y = f (2). (©) Ifthe graph of / is stretched vertically by a factor of 2, its equation becomes y = 2/ (x). 41, (@) Ifthe graph of / is shifted 2 units upward, its equation becomes y (b) Ifthe graph of / is shifted 2 units downward, ts equation becomes y = (h) Ifthe graph of / is shrunk vertically by a factor of2, its equation becomes y @) Ithe graph of fis stretched horizontally by a factor of 2, its equation becomes y = f (}2) @ Ifthe graph of Fis shrunk horizontally by a factor of 2, its equation becomes y = f (2x). —$$—$——— > ‘TRUE-FALSE QUIZ ————— 1. False, Let f(x) = and (= 1. Then f (s-+0)=(-14 1)? = SO+M= CH +P=240=Fe40. 2 False. Let f(x) = 32. Then f (-2) = =f Q, but 242. 3. False, Let f(x) = 27, Then f Gx) = Bx)? = 95? and 3 f (x) = 3x. So f Bx) #3, (8). 4. True. Ifxy £22). 5. True. See the Vertical Line Test 6, False, Let f(x) = 3? and g (x) = 2x. Then (f 0 g) (x) = f(g (x)) = f 2x) = (2x)? = 4x? and Ge NW) =8U W) =8 (7) =2? Sofogtgos. 36 CHAPTER FUNCTIONS AND MODELS EXERCISES ee 4 (a) When x = 2, y 2.7, Thus, f @) * 27. () /G)=3 + 182356 (©) The domain of f is ~6 < x < 6, 0r[-6. 6] (a) The range of fis 4 < y < 4, 0r[—4, 4} (©) F isinereasing on (~4, 4) (8 Fis odd since its graph i symmetric with respect to the origin. 2, (a) This curve fs nor the graph of a function of x since it fails the Vertical Line Test. (b) This curve is the graph of a fmction of x since it passes the Vertical Line Test. The domain is [~3, 3] and the range is [~2, 3] 3@) 4 vied 0 Tere ‘There will be some yield with no fertilizer, increasing (0 From the graph, we see thatthe dance Is ‘yields with increasing fertilizer use, a leveling-off of slightly less than 150 fet ‘yields at some point, and disaster with too much ferilizer use 5.) = V4=3, Domain: 43:7 20 > 3x2<4 = EG ysvi = 0sys2 a Domain: x+140 = x41. Range: all reals except 0 (y = 0 is the horizontal asymptote 2 Oand 6 ee forg,) 7. y= L+sinx, Domain: R, Range: “I Osys2 8, y =tan2x. Domain: 2x ¢ § ban => x # ¥+ $n, Range: the tangent fametion takes on all eal values, so the range is R. 9. (a) To obtain the graph of y = f (x) +8, we shift the graph of y = f (x) up 8 unit (€) To obtain the graph of y = Ge +8), we shift the graph of y = f(x) left 8 units. (© To obtain the graph of y = 1 +2/ (x), we stretch the graph of y = / (x) vertically by a factor of, and then shift the resulting graph | unit upward. (@ To obtain the graph of y = f(x — 2) —2, we shift the graph of y = / (x) right 2 units, and then shift the resulting graph 2 units downward (© To obtain the graph of y= —/ (x), we reflect the graph of » = / (x) about the x-axis, (f) To obtain the graph of y = 3 — f (x), we reflect the graph of y = f (x) about the x-axis, and then shift the resulting graph 3 units upward. 10. (2) To obiain the graph of p = f(r 8), we shift the graph of y = f (x) right 8 units, (© To obtain the graph of y reflect the graph of, x-axis, and then units upward. =F), we F (2) about the the resulting graph 2 M1, To sketch the graph of y = 1+ VEF2, we shift the graph of y = / left 2 units and up 1 unit, 13. To skeich the graph of y = cos3x, we compress the graph of y = cosx horizontally by a factor of 3 CHAPTER 1 REVIEW cl 57 (©) To obtain the graph of y = — (x), we reflect the graph of y = f (2) about the x-axis. (6) To obtain the graph of » = $f (2) ~ 1, we shrink = £9) by a factor of, and then shift the resting graph I unit downward the graph of. 12. To sketch the graph of y = (x ~ 1)" — 1, we shift the graph of y = x right 1 unit and dowa 1 unit. y 14, To sketch the graph of y= 3 — 2sin.x, we stretch the graph of y = sinx vertically by afactor of 2, ‘reflect the resulting graph about the x-axis, and thon shift that graph 3 units up (BI) 38 CQ CHAPTER FUNCTIONS AND MODELS 15. (2) The tems of / area minture of odd and even powers of, so i neither even nor odd. (6) The terms of fate all odd powers ofx, 30 fis odd 03 (Cx? 4 sin(-) 0s (x2) = (2), 50 f is even. 1 —sinx. Now f (—x) # f (x) and f (—x) # —f (=), so / is neither even nor 0-2 +2 y =—2x —2. The cirele has equation x? + y? -2x=2 if-2 x [0,x) (2x, 32], [-2n, —2}, (4, 5a], [-42, -32],..., 80 D = (x | x © [2nx, w + 2ne], where mis an integer). (go NG) =8 UF) =e (VX) = sin JX. x must be greater than or equal to 0 for /* to be defined, so D=[0,00). (fe Ae) =F) =F (v3) = VE = YF. D= 10, 00) (g og) (x) =g(g (x)) = (sinx) = sin{sinx), D=R. 18. Let h(x) =x + VF, g(x) = ¥¥, and f (x) = Ix. Then (f og oh) (x) ie 19. The graphs of f (x) = sin® x, where is positive integer, all have domain R. For odd , the range is {1,1} and for even 1, the range is (0, 1). For odd n, the functions are odd and symmetric wit respect tothe origin. For even rn the functions are even and symmetric with respect othe y-exis. As becomes larg, the graphs become less rounded and more “spiky”. 29. (a) Let x denote the numberof taster ovens produced in one -wook and y the associated cost. Using the points (1000, 9000) and (1500, 12,000), we get an equation of a 12,000 — 9000 1500 = 1000 y= 6(x = 1000) +9000 => y= 6x +3000. line: y — 9000 = («= 1000) = “0 1000 1500 2000 (eater ovens) (b) The slope of 6 means that each additional toaster oven produced adds $6 to the weekly production cost (©) The y-intercept of 3000 represents the overhead cost — the cost incurred without producing anything. 2 as Many models appear to be plausible. Your choice depends on whether ‘you think medieal advances will keep increasing life expectancy, or if there is bound to be a natural leveling-off of life expectancy. A linear model, y = 0.263x ~ 450.034, gives us an estimate of 76.0 years for the year 2000. 2m10 Principles of Problem Solving By using the area formula for atriangle, 4 (base) (height), two ways, we see that $ (4) (») = 4 (i) @), soa = 2 Since 4 + y? =, AVR y= VHT, anda p? — 100 2 from 4 times the area ofthe triangle. In this case, the area of the triangle is 2 POM 5 phar 2h = 2 Refer to Example 1, where we obtained ‘The 100 came $a (2) = 64, Thus, = oP p2 @ 2Ph+24h=P? = hOP+2*=P? => ha 2P +34 1-2. ifs <4 2x1 ite} 5 ifr <-5 oes x45) ifx>-S wei ssi-[ ‘Therefore, we conser the thre cases x < =5,~5 },wemusthave2x—1-@+5)=3 x=9 So the two solutions ofthe equation are x = ~J and x =9, lax ifx 7,whic is false If1 5 <> x > §, which is false because x <3, Itx > 3, wemusthave x —1—(—3)>5 e225, whichis false. All three cases lead to falsehoods, so the inequality has no solution, 8. £6) =|? — 4x1 +3), Ifx > 0, then f@) = fx? — 42 43] = (x ~ 1) @ —3)] Case (> MED 3, then f(x) =x? = 4x +3, enables us to sketch the graph for x > 0. Then we use the fact that / is an even function to reflect this part ofthe graph about the y-axis to obtain the entire graph, Or, we could consider also the eases x < —3,-3 y>0 © x-ytr-yps2 @ Case (ii: y>x>0 So yoxbecys2 @ 0<2 (11) Case (ii); x>0andy <0 @ x-ytxtys2 @ s2 @ Case (iy: x 0 @ y-x-x-y<2 @ -2es2 © Casey: vex <0 xnynertys2 & 02 (tr) Case (vi): x og et OTE 2A2 soy 322 7 (b) Using the points (0.8, 0) and (5, 118) from the approximate tangent fine, the instantaneous velocity at ¢ = 2is about HBE® = 28 fs. 9. For the eurve y = sim (10x /x) and the point P (1,0): @ 7]. 0 nro x @ mrp 2 120 0 os | 5,0) 0 18 | (5,086) | 1.7321 06 | (06,0.8660) | -2.1651 14 | (.4,-04839) | -10847 07 | @2,0.7818) | -2.6081 13 | (1.3, -0.8230) | -2.7433 08 | @8,1) = 12 | (1.2,0.8660) | 4.3301 0.9 | (0.9,-0.3420) | 3.4202 11 | (.4,-0.2817) | -2.8173 0.9 | (0.99,03120) | -31.2033 ‘As x approaches 1, the slopes do not appear to be approaching any particular value @) 1 ‘We see that problems with estimation are caused by the ‘requent oscillations of the graph. The tangent is so steep at | P that we need to take x-values much closer to 1 in order to os 2 get accurate estimates ofits slope {) If we choose x = 1.001, then the point Q is (1.001, ~0.0314) and m po ~ ~31.3794, [Cx = 0.998, then Q is (0.999, 0.0314) and mpg = ~31.4422. Averaging these two slopes gives us the estimate ~31 4108 === The Limit of a Function 1. Asx approaches 2, (x) approaches 5. [Or the values of f (x) ean be made as close to S as we like by taking x sufficiently close to 2 (but x # 2), Yes, the graph could have a hole at (2, 5) and be defined such that (2) = 3, 2 Asx approaches | from the left, (x) approaches 3; and as x approaches from the right, f(x) approaches 7. No, the limit does not exist because the lets and right-hand limits ae different. 4 0 CHAPTER2 UMTS AND RATES OF CHANGE (a im, £2) = oo means that the values of (x) canbe made arbitrarily large (as large a we please) by taking x sufficiently close to ~3 (but not equal to ~3). (©) im, # (2) = ~90 means that he values off (2) ean be made arbitrarily large negative by taking x sufficiently close to 4 through values larger than 4. 4. (a) lim F () ©) lim f= 4 © im, Fo (@ im, f (2) does not exist because the limits in part (b and par (care not equal OL@=3 5. (a) kim £() ) tim f(x) =2 ©) tim, £6) (@) Lim £ (2) docsnt exist because the limits in part (6) and part (6) ae not equal OF) lim se) @ tim, £6) = ©) tim) =—1 OFOD=-3 5 (lim _e@)= () tim, ee =1 (©) im, g (*) doesn't exist Oa © tim gts) =1 © tim, #) (g) jim. g@) doesn’t exist (h) gQ)=2 @ lim, g (3) doesn’t exist ©) tim 60) =2 (8 0) docso’t exist () lim ¢@) =0 7. (a) fig f (b) Jim #0) = © lim, f(s) =1 (8) im fe) =1 (© lim £6) =2 (© im, # (2) doesn’t exist because the limits in part (f) and part (e) are not equal. 8. (a) lim.e(x) =0 (©) tim g(a) = 00 © lim, g(@) = -00 © Jime@) = (€) The equations of the vertical asymptotes: x = ~5,x = 0,x 8. (a) lim f(x) = 00 © tim £6) (©) tim, £@) = (@ tim (= 00 ©) Lim J) = -00 (®) The equations of the vertical asymptotes: x = —9,x = —4, 10, im_F() = 180meand_ liz, 70) =300mg. These limits show that there is an abrupt change in the amount re a of drug in the patient's bloodstream at ¢ = 12h, The left-hand limit represents the amount of the drug just before the fourth injection. The right-hand limit represents the amount of the drug just after the fourth injection. nh. -2| SECTION 22 THELIMITOFAFUNCTON O47 @ tim so)=1 tim, 76) =0 (©) lim £ (5) =O does not exist because the limits in part (2) and | 7] 3 part (b) are not equal 12 ima F (x) exists forall a except a = +. 1B . : 7 ot ° i 18 Forg (x) = 5 16. For g(x) eO=5] 3. Fore x | 2) x | ee) x [ee | 02 | oxoeas2 | [ie [0.165503 | 3 =I 0.4 | 0.641026 | | 1.6 | 0.193798 | 24 4.8028 06 | osto04 | | 14 | 0.229358 | 201 | 43.368 08 | 0.409836 | | 12 | 0.274725 | 2.001 | —429.08 09 | 0.369004 | | 1.1 | o302iis | 20001 | —4286.2 0.99 | 0.336689 | | 1.01 | 0.330022 | 200001 | -42858 {eappears that fim It appears that lim, ir yar -10 48°C CHAPTER2 LIVITS AND RATES OF CHANGE ya) 3 1 For Fee) = HAAR 8 18. For F(t) x- 2 ~ LFoO x | Fe) 1 [Fo 26 | -o.0os8ea | [24 | 0.004124 Ts _ | 0.643905 255 | -o.003941 | | 245 | —o.004061 12 | o6s6sse 25.1 | 0.003988 | | 24.9 | 0.004012 11 | 0.661358 25.05 | —0.003994 24.95 | —0.004006 1.01 | 0.666114 25.01 | 0.003999 | | 24.99 | 0.004001 1.001 | 0.666611 appears that lim, F (x) = —0.004 Yet ats ea appear that in 7 cosx=1 19. For f(x) 20. Forg (a) = SES x | fo x | e@) 1 | 0.459698 1 | 0.546302 05 | 0.499670 os | -0.255342 04 | 0.493369 04 | -0.202710 03 | 0.496261 03. | 0.151135 02 | 0.498336 02 | 0.100335 0.1 | 0.499883 oa | -0.0s0042 0.05 | 0.499896 0.03 | —0.025008 o.01 | 0.499996 0.01 | -0.005000 cosx fon SEE appears tat lim —> Ie appears that Jim, 21 im, 2 = oo since (2-5) + 04s.» 5* and £5 > Ofors > 5 ~ ana £ 22 im moo since («= 5) Oasx 9 5° and 25 < Ofer <5 (23. lim, = 00 since (x — 3) > Oasx + 3 and >. 23 @ x x-1 - 28 tim AE = oo since x? > Oas x > O an <0 ford 2* and Far FED <0tt Oas.x + x~ and sinx > 0 for0 (-1/2)" and cosx < 0 for sary sera an 1 SECTION 22 THEUIMTOFAFUNCTION 49 2. (a) x fo) x £@) 05 =114 1S 0.42. 09 3.69 ua 3.02 099 | 33.7 Lor | 330 0.999 | —333.7 001 | 333.0 0.9999 | ~3333.7 Loot} 3333.0 0.99999 | ~33,333.7 00001 | 33,333.3, From these calculations, it seems that lim _f (x) at (6) If is slightly smaller than 1, then x* ~ 1 willbe a negative number close to 0, and the reciprocal of x3 — thatis, (2), willbe a negative number with lrg abcolte vale, So, im J) 20. fx is slightly larger than 1, then x? — 1 will bea small positive number, and its reciprocal, f (x), will be @ large positive number. So_ lim, F (x) =e. (© Mappears from the gph of f that lim. f(x) = ~9e and lim, f (2) = 00. at (-2@+1) x > 2+, the denominator approaches 0, and y > O for.x <—1 and 30. (@) » ‘Therefore, as.x > —1* or forx > 2,80 lim y= lim, y= 00. Also, asx — ~1> or a + 2°, the denominator approaches 0 and y < Ofor—1 3 Pax-12_ | &+30-4) rs os 42 a xetx-2 14 tin Zoe MN G@-DG=D i= 92 =25 _ (iP — 10h +25) ~25 h Sh 15. jim, im, 7 jim = 10) = -10 Bol GHD Hr +1) S4xth Peitt 3 16 aa epee EA ist 72 ar tim CE AMAA MPH AT dik OH 4 a 4 “pao i ai) i = tim ra = fing (64 6h 4? 4 tim BEIAAOM +I) —8 ah the +A? mm kA aig SOC-9 lim (04/7) =3 +9 =6 = jim, (12-4 6h +92) = 12 tim @43) 1435 193 FD) (C2) ~ THD Re + “1 (Aaa) Ba MH 16 e+ DE—D 244) eben? x= = +2) (244) 232 BB. ten 28! oe ji E=DEFD — jig VE= DYE +I +9) 0 Je-3 3 e-3 a vin} = lim (JF +3) +9) = lim (YF +3) him & +9) = (v5+3) O49) = 108 im (2) (8? 44) = tim (x +2) tm, (0? +4) “I G=peFy “aT VUE _ ji (= TFA) (1 VTA) =t Wet 50 Trel(evie) 0TH va) , 1 1 US viey EO ED 2 aan (; 1 8S 86 CHAPTER. UNITS AND RATES OF CHANGE 3-648) _ jim GtH-'=3" ‘on SOhG+ AS ms 26. Mies aa tim ME =4 = him 2 Mo? =a @—a 7b LE ll= va) (14 VE +8) l= ve sin [VE (14 VF-48)] = fog TLL DT Gfference of eubes) Another Mothod: We “add and subtract" n the numerator, and then split up the fraction: fa? (VF-I)+(l-2) a-90+%) ia y se om fe . (= va t+v90+9 jy [14 OE Det) 4 (a jarnas ae ts ® = 7®) =o00r | 0.686163 0.0001 | 0.6666167 - : 0.00001 | 0.6566617 0.090001 | 0.656662 nos 0.000001 | 0.666672 | lin x A 0.00001 | 0.666717 ax— 0.0001 0,.6667167, oot _ | 0.671663 “The limit appears tobe 3 Go tin (ERAN) — py TEE) _ , WTR) BN Jeera vise) OB Gaya t ae 4 im (JTF +1) (Limit Law 3) = 3 [yg + tn 1] ae ( (FTE +1) 387) =WFTI41) aa SECTION23 CALCULATING UMITS USING THEUMTLANS C87 2. @) os CO) = 76) =o.001 | 02886502 =o.0001 | o.288677s =0.00001 | 0.2886754 “1 ' =0.000001 | 0.2886752 ‘ 0.900001 | 0.2886751 tim BEM 9 9 0.00001 | 0.2886749 ~ o.0001 | 0.2886727 ot | o2esesi1 ‘The limit appears to be approximately 0.2887. (© tim (YSER= VS VEFE+ V3) _ i G43 tig 1 2 x Serre A) 0 ( sare) ea ti -— Limit Laws $& £ Rs tintiamse® 1 -— 1 oan flim G+) + V3 ‘ > 1 -—_. 8 Br res 1 “WS BA Let f(x) = —32, g (x) = 7 c0s 20x and A (x) = x2, Then 1 -1 ava Fx? < VASE sin(e (x) < VFR = £68) < #18) < h(x), Sosince tiny f(s) = lim (x) = 0, by the Squeeze Theorem we have lim, g(x) = 0. BLS $5) 5x7 420 +2 forall x, Now fim, 1 = Land elim, (7 42s 42) = lim x?-+2 him x + lim, 2= (1)? +2(-1) +2 = 1. Therefore, by the Squeeze Theorem, im, f() = 1 58 CCHAPTER2 LIMITS AND RATES OF CHANGE YW. 3x < f(x) $x +2 forO VE < Rll + sit Qn/s)] < 2VK, Sine tim, YE = Oand tim, 2/5 lim, [v¥ (1 + sin? @x/2))] = Oy the Squeeze Theorem. ), we have 37. Ih > —4,then be FAL =x44,50. lim, Ix +4l= lim, & +4) G44, “tn ka 444=0. Ifx < ~4, then px +4] tim Gs) =--444) soe Since the right and left limits are equal, tim, x +4 = 0. 38. Ifx < —4, then x +4] +4), 50_ tim = lim eq dod en Lif <2, then 38. Ifx > 2, then fr —2 bk-2 m G~ 290 Nim 2 AB aa AR ka =1. The tight and left limits ate be-21 disferent, 30 fin E=F does not exist (0,1 > 4, then 2x — 31 = 24 3,50 2x? — 3x 2x? — 3x x (2x — 3) fi FE tim 3 tim SRK D tim a = 15.1 px-3 lin, Broa = lite Bees = UM, aaa Ame bs a < Zhen 2x? — 3x ti 2x? — 3x im x (2x —3) lim Qx= 3) xons- = Qr 3) xan im =1.5. The right and Jf imi sas Be=3] 315 Fehr an * are different, 0 lim, “¥——+* does not exist aH 3 4. Since Ix ~xforx <0,vehie tim (£— 2) = tig (2-2) = tig 2, wien does not exis sine ano Ag 7) me Me the denominator approaches 0 and the numerator does not ~ a) walig (5-2) =. ee 42, Since [| =x forx > 0, we have tim ( B® (©) @ Since sen = 1 for > 0, tim, genx = im 1 Gi Since sgn s = = forx <0, tim sen x = tim ice im senx lim sens, lim, sgn.x doesnot exis. (iv) Since [sgn] = 1 forx 7 0, tim [sgn = im 1 SECTION 23 CALCULATING LIMITS USING THE UMIT LAWS O59 (in £¢4) = tim (9? -2 42) © x = lim x?-2 lim x + lim 2 Bat ae + a =Pa24 Hon, (6) = in, @— 5) = im, 3— im. x= 3—1 = 2 (©) fim, £() does not exist because lim f(x) tim, f(x) ml © 0) 0 tin PEP = ti St tg tym? © (0 in tal Spey = D8 (0 No i #4) does exis sein, re lin FO) 46. (a) @) tn, A(x) = lim, v=0P=0 {b) y (i ig He = lig + = 0,50 fim 4) = 0 ‘ iy 1) = fis Pet ; IN (iv) ig AG) = Nim a? =F = 4 ot + (0 in, 16) ig n=8- (i) Since im_h (x) ir, (x. im (x) doesnot exist 8.0) OP] =~2for~2. a isnot an integer. 48. (2) ©) @ im £8) = fim e— ED = tim bY) = at Gi tim, f(x) = tim, — © Jim FG) exists <> a isnot an integer 60 CHAPTER2. UMITS AND RATES OF CHANGE 49, The graph of f (x) = IE + I>; J (@) = 0 for any integer a. Thus, lim f(x) = Vand tim. fe £Q) = 121+ f-2]=2+ (-2)=0 oe ° Be (uJ a Jefl-hand limit is necessary since L is not defined for v > c. is the same as the graph of g (x) = —1 with holes at each integer, since $0 lim, f (2) = 1. oVT=T = 0. As the velocity approaches the speed of ight, the length approaches 0. A 51, Since p (x) is a polynomial, p(t) = ap + a1x + ax? +--+ + ax”. Thus, by the Limit Laws, dim, 2 () = fim, (ay + ax aya? +++ ay") ay + ay fit x + ap fim 3? +--+ fin x” Say + aya + ana ++ aga” = pla) ‘Thus, for any polynomial p, lim p (x) = p (a). 82. Let (x) = ee where p (x) and g (x) are any polynomials, and suppose that q (a) # 0. Thus, cy _ time) @ din (2) = fim 22) = LimitLaws) =2 (Exercise 51) =r =H Ge) = Tmgey MND = Gey xaos) =r @) 5B, Observe that 0 < f(x) < x? for all x, and is lim x?. So, bythe Squeeze Theorem, lim, f(x) = 0 54, Let (x) = [xD and g(x) = fx. Then lim f (2) and lim g(x) do not exist (Example 10) but Jim (7 @) +¢@)] = Lim (eI - BD = Limo =0. 55. Let f(s H (x) and g (x) = | ~ H (), where His the Heaviside function defined in Exercise 1.3.59. Thus, 7 (2) and fim g (2) do not exist, but sam) = fin (Vera = VS=RH1) _ 5 (Saxo 4 VIET “ih (pani e Veaa ea] en Saxo oma, (=) (V3=e+1) _ |, VS=R+1 “IR Gra (Veas+2) Roma 42 2 either 7 or g is 0 for any value of x. Then ‘Tim [7 @) 2 G01 = lin, 0 = 0. a) 57. Since the denominator approaches 0 as x -> ~2, the limit will exist only ifthe numerator also approaches 0 as x) 2, In order for this to happen, we need lim, (3x?-+ax-+a+3)=0 3-2? +a(-2)4a43=0 4 12-2a4a43 BHI HIB 34243) _ 3243) wt, Geyxsd x2 DEF 2 8. With a = 15, the limit becomes SECTION 24 THEPRECISE DEFINTIONOF ALIMIT. «61 58. Solution 1: First, we find the coordinates of P and Q as functions of. Then we can find the equation of the line determined by these two points, and thus find the x-intercept (the point K), and take the limit asr > 0. The coordinates of P are (0,7). The point Q is the point of intersection of the two circles x? + y? G-IP+y ex Eliminating y from these equations, we getr? =x? =1—(—1)2 2 4-2 Substituting back into the equation ofthe sri (eye (Viz F541) wow we take timitasy 08 tig x = im, 2 (Y= $9 +1) = in 2 (VT) So the limiting position of R is the point (4, 0) raat 0+ rot Soluion 2: We add a few lines to the diagram, as shown. Note that LP QS = 90° (subtended by diameter PS). So £S8QR = 90° = LOOT (subtended by diameter OT), I follows that COQS = ZTOR. Aiso LPSQ = 90F ~ LSPQ = LORP, Since AQOS is isosceles, so is GOT R, implying that OT = TR. As the circle shrinks, the point Q plainly approaches the origin, so the point R must approach a point twice as far from the origin as 7", that is, the point (4, 0), as ae . 1. ome = the Pre 1. (a) To have Sx +3 within a distance of 0.1 of 13, we must have 12.9 <5x+3<13.1 => 99<5x<101 => 1.98 29.99 < 6x < 30.01 = 4.9983 < x < 5.0018. Thus, x must be within 0.0016 units of 5 so that 6x — 1 is within 0.01 of 29, ¢ Definition of a Limit (6) As in part (@) with 0.001 in place oF 0.01, we obtain 0.00018. (6) As in part (a) with 0.0001 in place of 0.01, we obtain 0.000016. CHAPTER 2 UNITS AND RATES OF CHANGE 3. On the left side, we need |x — 2} < = 4. On the right sie, we need Ix ~ 21 < |#f — these conditions to be satisfied at once, we need the more restrictive of the two to hold, that is, Jy ~ 2] < $. So we ean choose 3 or any smaller positive number. 4. On the left side, we need [x — 5] < [4 — 5] = 1. On the right side, we need |x — 5] < [5.7 — 5] = 0.7. For both conditions to be satisfied at once, we need the more restrictive condition to hold; that is, x ~ 5] < 0.7. So we can choose 6 = 0.7, of any smaller positive number. the leftmost question mark isthe solution of / = 1.6 and the rightmost, / = 2.4. So the values are 1.6? = 2.56 and 2.4? = 5.76. On the left side, we need |x — 4] < [2.56 — 4] = 1.44. On the right side, we need |x — 4] < [5.76 — 41 = 1,76, To satisfy both conditions, we need the more restrictive condition to hold — namely, |x — 4] < 1.44, Thus, we can choose 6 = 1.44, or any smaller positive number. 6, The left-hand question mark is the positive solution of.x? = }, that is, x = 5, and the right-hand question mark is the postive solution of x? = 3, that is,x = y/3. On the lft side, we need [x ~ 11 < [45 ~ 1] = 0.293 On height side, werd = 11< |/f =| + 0.224 ounsng dawn to es), The more rei o these ‘wo conditions must apply, so we choose 6 = 0.224 (or any smaller positive number). 2, |VEFT 3) <05 2 2.5 < fax FT <3, Weplot the three prs of this inequality om the same sereen and identity the x-coordinates of the points of intrseston using the cursor. It appears that the inequality holds for 1.32 < x < 2.81, Sinee [2 = 132] = 0.68 and [2 - 2.811 0 <5 < min (0.68,081} = 0.68 81, we choose 8. |sinx — 3] <0.1 > 04 0) = 17.8412 em, 11786 <1 = 128888. [TBF — J ~ 004466 and fH ‘the radius within 0.0445 em of 17.8412, the area will be within 5 em? of 1000. (©) x is the radius, / (x) is the area, aris the target radius given in part (a), J. isthe target area (1000), is the tolerance in the area (5), and d isthe tolerance in the radius given in part (b). 1009 = 0.04455. So if the machinist gets 1M, (@) T= 0.1w? + 2.155w +20 and T= 200 => (b) From the graph, 199 < F< 201 => 0.1u? 4 2.135w+20= 200 => (by the 32.89 0) the target input power given in part (a), fis the aco, target temperature (200), ¢ is the tolerance in the temporatre (1), and isthe tolerance in the power input in watts indicated in part (b) (0.11 watts) as as (wats) 15, Given & > 0, we need d > O such that if [x — 2] <4, then IGx-2)-41<¢ @ Bx-6 (Gx —2) =A} < 8, Thus, fim (x —2) = 4 by the definition ofa limit 16. Given « > 0, we need d > 0 such that if lx — 4] <4, then [S-2x)-CO1 0, we need 5 > 0 such that if fe — (=1)| < 6, then [Gx48)-3) (Se +8) = 3] 0, we need 5 > such that if Jy — (DI < 6, then tee IB=4x)-I ce & [nde-dee oo dietllce @& el [r= (CDI < ¢/4. So choose 6 = #/4. Th M7- 1948) = 71 0, weneed 3> OsuchthatitO < jx —3] ° |x - 3] < Se. So choose § = Se. Then0 <|x-3} 0, weneed d > O such that iO < fx ~ 61 <4,then |(}-+3)—3| <0 e |¥—df [x6] Boo. 3 [ysl = +3) — 3] 0. weneedd> Osuehitarf x — C991 \(@-%) ce. Thos, im, ( -}) 7. by the definition of a limit, 4x12 iven 6 > 0, we need J > 0 such that if0 < |x ~ 3] < 8, then 0, we need 4 > O such that if |x — al < 6 then Jy ~a] < 2. Sod = 6 will work a ven > 0, we need > O such that if |x ~ al < 6 then je ~c| < #. But fe —el matter what 6 we pick. 0, so this will be true no B, Given: > 0, weed > Osuch that if|x| < Sthen x? =O] bby the definition of a timit. 26. Given © Then jx 0) <3 = 0, we need 3 > 0 such that ifr — 0) < 6 then IIx] — 0] < #. But |[xIl = b=. |. So this is true if we pick 65) CHAPTER2 LIils AND RATES OF CHANGE 28, Given: > 0, wenced d > 0 such that if 9-8 |Y=¥-0] 0, we need 3 = Osuch that if fx — 2] < dythen [(x?—4x-45)— 1] 0, we need d > Osu that if Wy ~ 3] Gerba > [rtdcd Soaked = min {1.2/8}. Then) =3] <8 > Kv=3)G +41 $ [83 =8- Le —3] <8) Se. So fi (2 4 = 4) = By hed jon of a limit BI, Given: > 0, we need & > O sue that if fx — (~2)1 < A then |(x? = 1) 3] < or upon simplitying we need x? =a] < ewhonever|y +21 <4. Notice that tix +2) < Lythen Tex $2<1 = —S fy = 2] © 5, Satake = min /5, I). Then fy ~ 2} < Sand [x 421 < /5. 50 (22 = 1) = 3) = HG 4 2p Ev — 291 = be 4 DIL — 2} < /5) (5) =e. Therefore, by the definition of a Limi firs, (2 <1) = 3 32, Given c > 0, we need 3 > O such that if by ~ 2] < d; then |x? — 8] f= 8) = fy 2 (x? 420 #4) < 19 =e, Soby the definition ofa limit, tim x* = 8. 38, Given # > 0, we ht) = min 2, §}. IFO < |e —3) O be given, We have to find a number 3 > 0 such that {S21 5, We find a positive constant C such that = ir BEB ep — jam ecanmabe fe ~2] Owe let 6 = min[1, 2x}. 160 < fr = 2] < then |x 2] <1 = Tov bat 226 Baxi . This shows toa Feed = jg) © 5 (osinpart Abo ir ~ 21 < 2r,50 yny= that SECTION 24 THEPREGSE DEFINITION OFA UMaTT 3. 1 Guessing a value for 6 Given e > 0, we must find & > 0 such that |Y¥ ~ Ja < « whenever O< Iya) <6. Bui Ye = Ya = ). Now if we can find a positive constant such that JE + Ya > C then < e,and we take [x = al < Ce, We ean find tis number by # n+ Ja Cc restricting x (0 fie in some interval Vit va> a+ Jas andsoc is bewal < (YFo- va)» Tis sugzes that weft = min [3 (Vier va) a] ntered ata. Ife al < Jauthen fa suitable choice for the constant, So 2 Shing tcl works Gene» 0, weltd~ min|Sa,(/fat Ya). 10 < al <3 en b-alcfo = eee eae: so _ We + Va) Ware Ja) 1 Given e = }, there exists 3 > O such that 0 < |e <6 = IHW@=Li<} @& Therefore, lim, JF = Jé by the definition of a limi. 36, Suppose that fim 1 (¢ L-$ $.Por-s<1<0HO= 1} <0 = <4}, Thivcontaies > J. Therefore, lim 1 ( does not exist 37. Suppose that lim, f (8) = FG there exists} > Osuch that 0 < |x| <3 = fC) L1 < }. Take any rational number r with < fr] < 8. Then f (-) = 0,30 0~ L| < 4,0 L < JE] < 4. Now take any irrational number s with O < [sf <6. Then f(s £ < $,90 lim £4) doesnot exis. 280 [I~ EL < $. Hence, 1—L < 4,50 > §. This contradicts 38, First suppose that lim, f(x) = L. Then, given ¢ > O there exists d > Oso that < |x—al <3 > IP) — Ll <6. Thena Sex 0< bal < S501 G)— LI O< Mal 0 be given, Since lim_f (x) = L, there exists 4) > 0.30 thata= 8) O sothat a a—d) 0, weneed 5 > Osuch that |r +31<8 > 1/(e43)"> M.Now os we 43) eee ererr i 1 HI < Te WHil< so fim G3! 68 CHAPTER 2 LIMITS AND RATES OF CHANGE 3 3 41, Let N< Obe given. Then, forx < =I, we have —— <, Sewtye @ Jeexthr Obe gi or hve SY VSO HD Yeeeah let (s 3 5 oe fe. men-t-o M+ t=e 00, there exists 41 > O-such that 0 < Ix —al <3) = Since fi 5 8G) =e, there exists ds > Osuch tha IgGe)—el g(x) > ¢= 1, Let be the smaller 05 and d:. Then 0 < |r =a) <3 = Fs) 490) > (M+ 1 = 6) + (= 1) = M. Thus, lim Lf 4) 4g G)] = 0 (b) Let A > Obe gi 0. Since Jim g (1) = ¢ > 0, there exists dy > O such that 0 < |x —al <0) = le) el €/2. Since fim F &) = 00, there exists bs > O such that O < fx ~ al < dp fea)» je tets=ins.9} Thad ME Jim £ (3) @ @) = 0. (©) Let N < O be given. Since lim g(x) = ¢ < 0, there ‘exists J; > O such that 0 < [xa] <3) > Ie) <-c/2 > 268) < 6/2. Since Jim, fe) = 90, thete exists bp > Ouch that 0 < [x — al <2 = f(x) > 2W/e. (Notethate 0.) Let d = min {é), 93). Then we O f&)>2N/e = fE)E@<—-F ¥, 80 fim f(x) g(x) = 00. =25 Continuity 1. From Equation 1, fin, (2) = £4. 2, The graph of doesn’t have any holes, jumps, or vertical asymptotes. 3. (a) The following are the numbers at which / is discontinuous and the type of discontinuity at that number: —5 (jump), ~3 (infinite), —1 (undefined), 3 (removable), 5 (infinite), 8 Gump), 10 (undefined), (b) 7 is continuous from the let at —§ and ~3, and continuous from the right at 8. It is continuous from neither side at 1, 3, 5, and 10, 4. g is continuous on [~6, 5], (5,3), (3, ~2), (-2, 1). (1,3). 13,5}. and (5, 7]. 5 6. SECTION2S CONTINUITY 68 7. (a) (b) There are discontinuities at ¢ = |, 2, 3, and 4. A person parking in the lot would want to keep in mind that the charge will jump at the beginning of each hour. 8. (a) Continuous; at the location in question, the temperature changes smoothly as time passes, without any instantaneous jumps from one temperature to another. (b) Continuous; the temperature ata specific time changes smoothly as the distance due west from New York City inereases, without any instantancous jumps. {€) Discontinuous, as the distance due west from New York City increases, the altitude above sea level may jump from one height to another without going through all of the intermediate values — at a cliff, for example. (@) Discontinuous; as the distance traveted increases, the cost ofthe ride jumps in small inerements, (©) Discontinuous; when the ights are switehed on (or off, the current suddenly changes between 0 and some ‘nonzero value, without passing through all ofthe intermediate values. This is debatable, though, depending on ‘your definition of current 9 Since f and g are continuous functions, im [2 (8) ~ x (8)] =2 lim, 2) ~ him, @ (by Limit Laws2 & 3) =2/ G)— £8) (by continuity of / and g at x = 3) =2:5-gG) =10-¢6) Since it is given that Jim [2 (x) ~ g (x)] = 4, we have 10 = g (3) = 4, or g3) =6. 10. Fin, £6) = fi, (2? + T=B) = tim? [lin TT = B+ VIRB = 16+ V3 =f By he definition of continuity, f is continuous at a = 4. 11 fin, £6) tim, (422)! (x2 fi ») eben lim, 3)! = 81 = f(-1). By the definition of continuity, / is continuous at a ceed tims t tina 441 12, Tim ¢ () = Jim, 2 (4). So g is continuous at 4 18. Ford ite 1B Aiscontinuous at 1 beea discontinuous at | sinee f (1) is lim f (x) does not exist. not defined. er WS | is discontinuous 18 Since f(x) = forx 1, we have ‘at —1 because f (—1) is not defined. ft, FG) = tim, x4 lim, & -)=-2. Bu SD = 6,80 fim, £4) # LCD. Therefore, Fis | diszominvost I a] : 18, Since f(x) al
  • 0} = (1 lil < 3) =[-5, 5}, Abso, 2¢ is continuous on R, so by Theorem 4 41, £0) = 2 + V25=F is comtinvous on its domain, which is (~5, 5] 28. e(x) =x ~ Land G (x) = x — 2 ate both polynomials, so by Theozem $ they ate continuous. Also f(x) = is continuous by Theorem 6, s0 f(g (x)) = &/x — 1 is continuous on R by Theorem 8. Thus, the product (x) = YE=T (x? ~ 2) iscontinuous on R by Theorem 4 4. ‘Fis continuous by Theorem 7. 1 whieh is 24, By Theorem 7, the trigonometric function sin.x and the polynornial function x + | are continuous on B. By Theorem 4, f(x) = = is continuous on its domain, (x |x 4 —1}, 25. By Theorem 5, the polynomial 1 ~ x? is continuous on (—c0, co). By Theorem 7, cos is continuous on its domain, R. By Theorem 9, cos (1 ~ x2) is continuous om its domain, which is R 26, By Theorem 5, the polynomial 2x is continuous on (00, 0¢). By Theorem 7, tan is continuous at every number in its domain; that is, {x |x # § +n). By Theorem 9, tan2x is continuous on its domain, which is fe 12x 4 $ +n} = {x |x x § + Fn} (the odd multiples of §). 27. By Theorem 7, the root function J and the trigonometric funetion sinx are continuous on their domains, [0, 00) ‘and (00, 00), respectively. Thus, the product F(x) = JX sinx is continuous on the intersection of those domains, (0,00), by Theorem 4 #4, 28. The sine and cosine functions are continuous everywhere by Theorem 7, so F (x) = sin (cos (sin x)), whieh is the composite of sine, cosine, and (once again) sine, is continuous everywhere by Theorem 9, 1 FE} _ ‘0 is undefined and hence discontinuous when sins Ttsinx =0 €9 sinx=-1 eo x=—§ 42an.manimeger oi} 0 30. 5 5 1 1 B= 200 5 a The funetion y trigonometric funetion and a root function, The square root function has domain (0,00) and the tangent function has domain {x |x # $ +n). So f is discontinuous when x 3.00, Since ~0.44 <0 < 3,00, there isa number cin (0, 1) such that /(e) = Oby the Infermediate Valve Theorem, Thus; there ia roo of the equation tanx = 2x = 0, or tan = 2x. in th interval (0.1, 4. (9) (6) = sing =2 +. is continuous on [0.2]. £0) thore isa mumbere in (0,2) sub that £6) equation sinx ~2.4¥ = 0, or sin ince by the Intermediate Value Theorem. Thus, and f (1) = cost — 1 —0.46, Since O by the Intermediate Value Theorem. Thus, there is 1 or cos x = x, in the interval (0, 1). 2 is continuous 2.and f (2) = sin2¥ O91. Since ~2 < 0 < 0.91, O by the Intermediate Value Theorem. Thus. there is a r001 of the 2—x, im the interval (0,2), (b) F110) © =0.009 and (1.11) * 0.006, so there is root between 1.10 and 1.1L. 74 © CHAPTER? UMITS AND RATES OF CHANGE 8. (a) f (x) = x5 — x7 + 2v +3 is continuous on [=1, 0}, f (-1) = =I. and f (0) = 3. Since -1 <0 < 3, there is a number c in (—1, 0) such that / (c) = 0 by the Intermediate Value Theorem. Thus, there is a oot of the equation x5 =x? 4 2x +3 = in the interval (1, 0) (b) (0.88) © -0.062 and f (0.87) ~ 0.0047, so there is a root between —0.88 and ~0.87, 48. (a) Let f(y = 9? = 4. Then f= = P= 4 $2 4 = 24> 0. Soby the Intermediate Value Theorem, there is a number ¢ in (1,2) such that ¢* — ¢? 4 = 0. (b) We can sec from the graphs that, correct to three dk imal places, the root is x © 1.434 | < and £ (6) = § > 0, and J is continuous on (5,00). So $0. (a) Let f0) = VES Ls. Then 05 by the Intermediate Value Theorem, there is a number c in (5,6) such that f (c) = 0. This implies that 1 = en3 er3 (b) Using the intersect Feature of the graphing device, we find that the root of the equation is x = 5.016, correct to three decimal places. S51. (=) IF f is continuous at a, then by Theorem 7 with g (1) =a ++, we have 1( fin, (a+ ») F(a). (eyLete > 0. Since fim f (a+ 1) = f(a) there exists 5 > O such that |i] <3 => im fa+h [flat hy= fl < ©. Soit heal 0, the interval 1 its irational (a ~6,a +0) contains oth infinitely many rational and infinitely many irrational mambers. Since f(a) = 0 or there are infinitely many numbers with fs — a] < Sand Lf (3) ~ f(@)l = 1. Thus im, (3) # F(a. Cn fast, Jim f (8) docs not even exist] 0 ix is cational 56. ex) = | - ite ivirationay SCOuousatO fo see why, note that ~ |x| < g (x) < [| so by the Squeeze ‘Theorem lim g (x) = 0= g (0). But is continuous nowhere ese. For ifa x O-and 5 > 0, the interval (a= 5,048) contains both infinitely many rational and infinitely many irrational numbers. Since g (a) = 0 oF a, there ae infinitely many numbers x with x ~ al < Sand |g &) —g (a)l > fal /2. Thus, fim, x (3) # g (a) ‘57. If there is such a number, it satisfies the equation xP +41 =x <2 x? x10. Let the LHS of this equation be called f (x). Now f(-2) = 5 < 0,und f (1) = 1 > 0. Note also that / (x) is a polynomial, and thus continuous. So by the Intermediate Value Theorem, there is a number ¢ between ~2 and —I such that /(e) = 0, sothate =< -41 88. (@) Hie, F (2) = Oand tim F (x) = 0, 50 lim (2) = 0, which is F (0), and hence F is continuous atx = @ if Fora > 0, fim F(x) = Jim x= a= F(@). Fora <0, fim F (2) = him (=x) = ~a = F(a). Thus. Fis continuous at x = a; that is, continuous everywhere (b) Assume that fis continuous on the interval /. Then fora € /, lim LF (x) Him, (| = IF a) by Theorem 7, (Ifa is an endpoint of /, use the appropriate one-sided limit.) So || is continuous on / 1 ifx>0 (6) No. the converse i false. For example the faetion f(x) =} 1 | i," isnot continuous atx = 0, but If ©)1 = 1 is continuous on R, TSG. GHAPTER2 UMTS ANO RATES OF CHANGE 59. Define w (¢) to be the monk’s distance from the monastery, as a function of time, on the first day, and define d (¢) to be his distance from the monastery, as a function of time, on the second day. Let D be the distance from the nformation we know that « (0) = 0, « (12) = D, d (0) =D and d (12) = 0. Now consider the function w — d, which is learly continuous. We calculate that monastery to the top of the mountain. Prom the gi (w= d) @) = ~D and (u ~ d)(12) = D, So by the Intermediate Value Theorems, there must be some time between 0 and 12 such that (uv —d) (fo) =O > ello) = d (). Soat time fo aer 7:00 A.M. the monk will be atthe same place on both days Tangents, Velocities, and Other Rates of Change s: mpg = Y= LO=LO) vmre = Ny x3 1. (@) This is just the slope of the line through two poir £6)-f@) (6) This is the limit ofthe slope ofthe sseant line PQ as © approaches P: m = jim f@+h) ; (b) Instantaneous velocity = fim 3. The slope at D is the largest positive slope, followed by the positive slope at E. The slope at C'is zero. The slope at Bis steeper than at 4 (both are negative). In decreasing order, we have the slopes at: D. E, C, A, B. 4, The curve looks more like a line as the viewing rectangle gets smaller, 2 po eae as 2 1 ~ 5 @) (ym LEVRLED _ jig, CP F2)—B) «gg EF IENY _ tim ety in, Joey lis Sey Talis gs ls =-4 fm Lith - S(-3) [(-3 + ay 4+2(-3 + )] - @) fi fim 2 = lim 2 6h +h? —~6 424-3 tim 2 4) lim (4 i i fo (6) The equation ofthe tangent line is © 4 4x + 3)ory = ae —9, SECTION 26 TANGENTS, VELOCITIES, AND OTHER RATES OF CHANGE =O 77 LOV=FEM jg BED _ jy OE DOR 241) ) cet eed ae x41 6. (a) (i) m= lim, = tim, 6? 4 0 Hen —se mee peri (i) m = fim FOE LD CLAW A ED jig HIP 4h THT ah * of A i) i = Jim (22-38-43) = = [im @? = 38-43) )y-CD=3e-CD) = ya3e42 7. Using (1), = lim LO=L@ _ yy (La 2x=3"*)- CI _ ~3x? - 2x 48 (23344) +2) m= lim = lim = = im Myce ANB aed) aa eg aD = lim, (3x +4) = 10 Thus, an equation ofthe tangent is y 47 = 10 (x +2) ory = 10x + 13. Alternate Solution: Using (2), atin LEFMA LO) _ jig, LOPHM = S172) _ jig, [= 224-3247] - 0 h ho a Shao A tg BRAM DAT tht 10) Sa fit 9 + 10) = 10 - Thus, an equation of 1 a 8. Using (1), m = fim the tangent line is y — 9. Using (1), m = lim, ~ 2/ +3)-2/(@ +3) 2(a-x) -2 11, @) mm Jim 27 EAD AEH _ jpg —2@ = 9) gg =? @m= jin xa I G=OeFN@FD METDUPD GES = 2 jiya=0 > m= =? ° ory ) @a= Gia TR © CHAPTER? {IMTS AND RATES OF CHANGE 12. (a) Using (1), = lim m= ti ta (tata) a yo@ SOON ing epam tice =o mat42C4 42(-)e0 Gi)y 1 meth2y=3 &) @« 13. (a) Using (1), = sim 1 so.an equation of the tangent ine (@) 4. x-LAtQD: (b) ACU, =2): mm = 3 (0) =-l0-) 8 isy im =3(2)? —4-~ 8, so an equation of the tangent ine is 14. (a) Using (1), m= im = jin == STE me I @ =a) VS= Be VB= a 7 2(x~a) = 18 Wray VBSRIC= i (VSS a Ba) 2 lim ——> "ix av ve D(VS—2a4 VS—3x) 2(S— 2a)? =09- 2a)? (b) AL QDs m 2a)? o (e=2) te) years ba (-2 {mas 2-IPM=g © ] =O oo veya SECTION 26 TANGENTS, VELOCITIES, AND OTMERRATES OF CHANGE 79 15, (a) Since the slope of the tangent at s = (b) The slope of the tangent is greater at C than at B, so the ear was going faster at C (c) Near 4, th is steeper us.x inerenses, so the velocity Was increasing, so the car was speeding up. Near B. the tangent lines arc becoming less steep, so the car was slowing down. The steepest ‘tangent near C is the one at C, so at C the ear had just finished speeding up, and was about to start slowing down. is 0, the car’s initial velocity was 0 ngent lines are becor (d) Between D and £, the slope of the tangent is 0, so the car did not move during that time, 16. sistance in ie 200 ath + 204 a> slope = 65 Wrath Wie 70 ope = 0 ‘lope = 85 ym yao rob 128 Let a denote the distance traveled from 1:00 t0 1:02, 6 from 1:28 t0 1:30, and ¢ from 3:30 to 3:33, where all the times are relative to t = 0 atthe beginning of the trip, 17. Lets () = 400 — 1602 im GOR N6)~ 16 -8-Dar—D a t=2 ma r=2 9) = tin O=5 2) = ~8 im (21 1) = 24 ous velocity when ¢ = 2 is ~24 fis. HO+h-H 7 (58 + ssi 18, (@) (1) = Jim, = Sim ey Math (©) »(@) = Sime i = ton EBAESSH ao = jim ($8 ~ 1.66a ~ 0.838) = $8 ~ 1.66a m/s (6) The arrow strikes the moon when the height is 0, that is, 8 ~ 0.83? =0 <> 1 (58 — 0.831) = 1 = gh ¥ 69.9 s(since can't be 0). (©) Using th ime rom pre» (%) = 58 1.66 (85) =58 m/s 58 m/s. ‘Thus, the arrow will have a velocity of 19. (a) 3) GHAPTER2 LIMITS AND RATES OF CHANGE (4a° + 60+2) a+ M=s0@) _ 5, MAA? + 604m) + > i ry i 4a 4 12a2h + Lah? 44H + 6a 46h +2 — 4a — ba 2 ms a pn, 2 Naa a [im (1222 + 12a +4? +6) = (12a? 46) m/s So (1) = 12 (1)? +6 = 18 m/s, » 2) = 122)? +6 = 54 m/s, and v3) = 123)? +6 = 114 m/s. 2, (a) The average velocity between times ¢ and 1 +A is sys) _ UH? CH= h +a +h 81 h =r+h—8) m/s @B.4k 1 =3,4 =4—3 = 1, sothe average velocity 526) + 1-8 = =1 mys (i) [5,4]: €=3.5, b= 0.5, s0 the average velocity 182.5) +05-8=-0.5 m/s. i) [4,1 = 4 = 2y+1-8 (iv) 4.45]: ¢= 4, = 0.5, so the average velocity is 2) +0.5-8 = 0.5 m/s |, $0 the average velocity is m/s, 21. The sketch shows the graph for a room temperature of 72° and a refrigerator temperature of 38°, The intial rate of change is greater in magnitude than the rate of change after an hour. BA+18 84H) + 18 (? 8 + 18) 2481-18 2th +h? 8h h sth~sO a = fim (21+ = 8 ard (6) 0) = fim, =2-8, son (4) =0. 2. The slope of the tangent (that is, the rate of change of temperature with respect to time) at 1 = Th seems to be about Bgl = -0.7PF ymin, nw SECTION 26 TANGENTS, VELOCITIES, AND OTHER RATES OFCHANGE 1 79-11 2. (a) (i) [8 10} -1.2%h n (Gi) {8,10} =1.257/n 10, (i (8,9; SASS (b) Inthe Ogure, we estimate 4 to be (18, 15.5) and B as (23, 6), So the 6=155 =1234h ==1.99h at P1995) = P (1991) _ 839-793 _ 46 40) 0 ~T595= 591 7 a 1 thousand peopie per year 839-820 19 ii) 8 lo sen leper yer () 5551585 9.5 thousand people per year 874 35 . (i) FypF—Tp9s = FT 17-5 thousand people per year IS+ITS (6) Using the values from (a) ans (ai), we have —— 13.5 thousand people per year. 0 Estimating 4 as (1993, 816) and B as (1997, 862), the slope at 862-816 1995 is [555 gg = HS thousand people per year isan AC _ C(105)—C (100) _ 6601.25 ~ 6500 _ 0) OS = gs = 2025) ait, ac C400) _ 6520.05 ~ 6500 _ we 0 = 20.05 (oy L1H) = C40) _ [5000-4 10 100-4 #) +0.05 100-4 1)?] 6500 _ 200 + o.0sH? i h =20+005h,h #0 C00 + A) ~ C100), So the instantaneous rate of change is fim, i = fim @0 + 0.051) = $20 unit 2 G_GHAPTER2_LIVITS AND RATES OF CHANGE 2 AV vEth-VO 0 ith +h? te )]_ = 1nora[(1 30 + 5600 30 * 3o00) |= 60, 120-4 26+ A) = BY (120-4 26 +A) Dividing AV by h and then leling h > 0, we see tha the 2 0,000 (1 - E#)— 10,00 ( h 2th re 30 * 3600 san) 5 4 — 60) envmin. Flow ate (galinin) | Waerrovainna P(e | o> ams 700,000 10 27777 69,4449 20 -2222.3 44,4449 30 1666.6 25,000 40 IT, Uk 50 - 5555 27777 S 0 ° ‘The magnitude of the flow rate is greatest atthe beginning and gradually deerenses to 0, =2 Review — CONCEPT CHECK ———$—$— 4. (2) lim, f ) = Ls See Definition 2.2.1 and Figures 1 and 2in Section 2.2 (6) lim, f (x) = L: See the paragraph after Definition 2.2.2 and Figure 9(b) in Section 2.2 (0) lim_ f(x) = L: See Definition 2.2.2 and Figure %a) in Section 2.2. (@) lim £ (2) = 00: See Definition 22.4 and Figure 12 in Setion 2.2. (©) fim £ &) = ~00: See Definition 22.5 and Figure 13 in Section 22. 2, See Definition 2.2.6 and Figures 12-14 in Section 2.2. 3. (a)—(g) See the statements of Limit Laws 1-6 and 11 in Section 2.3. 4, See Theorem 3 in Section 23. 5. (@) A fiction Fis continaous at a number aif f(x) gots closer to f(a) as.x gets close to a, that is, jim 1@) = f(a. (8) A function / is continuous on the interval (~00, 20) if fs continuous at every real number a. The graph of sueh a function has no breaks and every vertical Line eroses it 6. Sce Theorem 2.5.10. 7, See Definition 2.6.1 8 See the paragraph containing Formula 3 in Section 2.6. 9, (@) The average rate of change of y with respect to x over the interval [x1, x0] is (b) The instantaneous rate of change of » with respect to x atx = xy is lim CHAPTER? REVIEW O83 —_—_—_—_—_—_—_———- TRUE-FALSE QUIZ ———<—<— 1. False. Limit Law 2 applies only ifthe individual limits exist (these don’t.) 2 False. Limit Law 5 cannot be applied ifthe limit ofthe denominator is 0 (it i.) 3 True, Limit Law 5 applies, 4. Truc. The limit doesn’t exist since (x) /g (x) docsn’t approach any real number as.x approaches 5. (The denominator approaches 0 and the numerator doesn’t) 2 xt - 5x or tig S225. oy sample in ssn 22,8 row hr hee Ht sts (and itis equal to 1) 5. False, Consider ty 6. False, Consider lim Lf) g (0) im [oe -0) = ] exits is aes bat 6) = and (6 does notexist 90 (ORO #1 7 True. A polynomial is continuous everywhere, so lim p (x) exists and is equal to p (6). Fae, conier i 10) 601= J (42h) thins oo Otte functions approaches 50 | Venn text 9. False, Consider £0) =| ae 10, False. The function f must be continuous in order to use the Intermediate Value Theorem. For example, let 1 if0 0} = (ce, ~3] U3, 00). Note that x? — 2 0 on this set and so the quotient continuous on {x |x? fimetion g(x) is continuous on its domain, (00, ~3]U [3, 00) 2 f(x) = 2x3 + x? 4 2is a polynomial, soit is continuous on [-2, ~1] and f (-2) = -10 <0 <1 = f(-1). So =I) sueh that / (c) = 0, that is, the equation by the Intermediate Value Theorem there is a number ¢ in (~2, 2x3 4x? 42 = O has a root in (=2, =1). 2B, Let f(x) = 2sinx — 3-4 2x. Now f is continuous on [0, 1] and f (0) = -3 < O und / (I) =2sin 1 = 1 0.68 > 0. So by the Intermediate Value Theorem there is a number ¢ in (0, 1) such that J (e) = 0, that is, the equation 2 sin x = 3 — 2x has a root in (0,1), 28. (a) The slope of the tangent line at (2, 1) is L@ ty 2x +2) 2 Tene y=? im, -2 +2) (b) An equation of this tangent line is y — | = —8 (x — 2) ory = ~8x +17, with x-coordinate a, we have 30, For a general p mn tim 220739 2/1 3a) _ 4, _2(1 3a) = 241 = 35) a x-a xmva (1 = 3a) (I= 3x) (x =a) tig 8 = 2) _ gu, —__ 6 __6 ste (= 3a) (T= Sx) (ea) 9 (13a) (I= 38) (1 3a)? Fora =0,m 0 an equation ofthe tangent line is y - 2= 6 (x ~ 0) or y = Ox +2. For Gand £0) Lom =f and f (1) = §,30 an equation ofthe tangent ine fs» — } (tory a det} BA. (a) = 1 421-4 7/4. The average velocity over the time interval [1, 1+ A] is s(t ay—sC) 142 +a) + (EAP 4 = 13/4 - h ~ A ~ intervals the average velocities are: () {1,3 (0+ 2)/4 = 3 mys (ii) 1,15]: (10-4 0.5) 4 = 2.625 m/s (i) 12k (0+ N/A =2.75 mys Gv) [1 1.1}, (10-4 0.1) /4 = 2.525 m/s sath) h (b) When ¢ = 1 the velocity is tim SCFM SD fy WAM 9 Sys Mh ms 32. (a) When I’ increases from 200 in? to 250 in}, we have AV’ = 250 — 200 = 50 in’, and since P = 800/V', 800 _ 800 P= — P2000) = s— - -4=-( fin’. the average rate of change is AP = P(250)~ P (200) = SO = TE = 3.2— 4 = =08 Ibyin*. So the average rate of chang AP ~08 Ibfin? CHAPTER? REVIEW © 7 () Since 1” = 800/P, the instantaneous rate of change of V with respect to P is sim, BY = iy HAP AMY= VAP) _ i 800/ (P+ hy = 800/P pb AP pb A ~ 7h = fim S201 — (PON _ 5, 800 800 MO hP+HP Hb +hyP~ PP which i versely proportional (0 the square of P. rea 33, The inequality < 0.2 is equivalent to the double x41 x1 y= [lx + 1)/( ~ Dl and y = 3.2 on the interval [1.9, 2.15}, we see that the inequality holds whenever 1.91 KO) < £6) < e(8) and fim es) Jim soy =0 36. (a) Note that / is an even function sin ‘any integer [a + [=n] A which is not an integer, Wd + (4) = ed + (- 1 S£@) = (=x). Now for |, and for any real number L. So fim £ (x) exists (nd is equal to =1) forall values of a. (b) / is discontinuous at all integers. Problems Plus 1. Let Vs,s0x = 15, Thent + Lasx > 1,50 tim ER i FA! a ug ERG HD jig Ft VET OM BAT GDP 4al) Siebel Pete 3 Anoter Method: Multiply both numerator and denominator by (VF-+ 1) (YE+ YR +1) JaeFb+2 arth 2. First rationalize the numerator: im LO EE? A tim Ot Now since the oO x Vax +2 10x (Var Fb +3) denominator approaches 0 asx — 0, the limit will exist only ithe numerator also approaches 0 as x > 0. So we require that a(0) + -4=0 = b=4. So the equation becomes lim 7 _ = “ 0 Javea h2 Vis? > a=4, Therefore. 3, For—3 0,50 [2x — 1] == (2x = 1) and [2x + I] = 2x 41 0. Asx 00, 1 00,80 Oand 1 by 6 (a) + Sinee (LxP and fv iP are positive integers or 0, there are only 4 cases: Case (i t= lox <2and0 The only integral solution of n? — £1. So the graph is and mt {eo Lh = 42, DD [psx s3or-2ex Isxty<2or DI =1-n = =n sy <2—n. Choosing integer -l lim x? = tim @+N > eat] > a= or -0.618. 8. (a) Here are a few possi (b) The “obstacle” isthe line x = y (see diagram). Any intersection of the graph of f with the line y = x constitutes a fixed point, and if the graph of the function docs not cross the line somewhere in (0, 1) thi ‘must either start at (0, 0) (in whieh case 0 isa fixed point) or finish at (1,1) (in which case a fixed point), (©) Consider the function F (x) = f (x) —x, where f is any continuous function with domain (0, 1] and range in 0, 1]. We shall prove that / has a fixed point Now if 0) = 0 then we are done: / has a fixed point (the number 0), which is what we are trying to prove So assume J (0) #0. For the same reason we ean assume that (1) #1. Then F (0) = f (0) > 0 and F (1) = f (1) ~ 1 <0. So by the Intermediate Value Theorem, there exists some number in the interval (0, 1) such that F (c) = f(e) ~¢ = 0. So f (¢) =e, and therefore f has a fixed point 9. fim £4) = im ($17 (0) +81 + 4 1/9) = 8(0)]) =F him) +26) +4 lim 6) 2) 24 feta fand fim, g ) = Jim 7 6) +869] ~ FO) = Hilf) +8 0) fin, (6) = 2-9 So Jim LF) (991 = [tim | [tie 00] = 4-4 = 3 PROBLEMS PLUS OSI Another Solution: wee Jim Lf (x) +. (2)] and Lim [f (x) ~ g (x)] exist, we must have nL to) +8 COF = (Jim L7G) +e GON)” ad Jim (70) = 8 OOP = (lim LO) - 209) +50 lim, U®) e@l= tim, Jim 4 (Lf) +2G)P - 1G) —gG)F) (because all ofthe /? and g? cancel) = # (tim, 1/ 6) + ¢ GOP = fim 17) = « GP) 10. (a) Solution J, We introduce a coordinate system and drop a perpendicular » fom Paes. Wes fam NCP ttn 20 = 2 ad em ZN BP that tan 0 = y/x. Using the double-angle formula for 2iand __20/x) tangents, we get Sa) Toa tan20 After a bit of sim 2 x Gr ~ 2). As the altitude 4M decreases in length, the point will approach the x-axis, that is, y > 0, so the limiting location of P must be one of the roots of the equation x (3x ~2) = 0, Obviously it is not x (the point P can never be to the Iefl ofthe altitude 44M, which it would have to be in order to approach 0) so it must be 3x — 2 =O, that is, Solution 2. We add a few lines to the original diagram, as shown. Now note that £BPQ = ZPBC (alternate angles; OP || BC by symmetry) and similarly ZCQP = LOCB. So ABPQ and ACQP ate isoseete and the line segments BO, OP and PC are all of equal length. As |AM| => 0, P and Q approach poi ison the base, and the point P is scen to approach a position two-thirds of the way between Band C, as above, (b) The equation y* = x (3x ~ 2) calculated in part (a) isthe equation of " the curve traced aut by P. Now as |AM| -+ 90,20 + 5,0 §. A x land tand = y/x,y > 1. Thus, P only races out the part ofthe curve with 0 < p< I (@) Consider G (x) = T (s + 180°) ~ T(x). Fix any number a. IG (a) = 0, we are done: Temperature at a = Temperature at a+ 180°. If G (a) > 0, then G (+ 180°) = T (a + 360") = P (a+ 180°) = P (a) — T (a + 180°) = ~G (a) < 0. Also, G is continuous sinee temperature varies continuously. So, by the Intermediate Value Theorem, G has a zero on the interval [o,0-+ 180°] 1FG (a) < 0, then a similar argument applies (b) Yes. The same argument applies. (©) The same argument applies for quantities that vary continuously, such as barometric pressure, But one could argue that altitude above sea level is sometimes discontinuous, so the result might not always hold for that quantity BLANK PAGE me: Derivatives =31_Derivatives The line from P to Q is the line that has slope J2+h)-$2) LO+h=FQ) f+) 2. Ash decreases, he line PQ becomes steeper, sits slope increases. So £M-f2 _ {B) @) fe)-~ £2) ase ST 3aa hh = sah = 20? = fim mT ~4a fin U2 tO +3 (044) ur@ i) how h Sab + Sah? 4 3 > > > = tm ee = firm (3a + 3ahh + 1? +3) = 30° +3 ath L@+W-F@ _ 5, FED a) mb h (4M Qa~N~aQa+Ih—N) 4 )@a= 1) $83 i @a4 2h - a= 1 15. /"(a) = jim at ES) 1 = tim 1) @a42h— Na) ~~ Ga-1P 16. f(a) = fim, Lari fo h(-a® = 1 = ali) at aah + i 9% 0 CHAPTERS DERIVATIVES fa Mb Rs tat hea (nat VID 2 Wo (esa hv3—a(/3-a4 vasa A) 2 1 * Ve=aV3—aQV3—a) Gay? Lath fla i, ETH Ve 3a 18. f(a) = jim, b ay (aE IET rn at 1+ Vari hi(aat ant 1+ Jia FT) = jim O24 34+ = Ga +1) ~ mb Vat ah + 1+ Jaa i) 3 3 fit, oa wh f3a+3h+1+V3a41 2V3a41 Note thatthe answers to Exercises 19-24 are not unique (em 19. By Equation 1, tin, = J" (I), where / (x) = VE. Or: f O), where f(x) = VTFE Q+h-8 h 29, By Equation 1, fim = J" (2), where f(x) 21. By Equation 3, tim ei eol SEH} 7G), where 6) = 008% f' (1), where f (x) = 2°. 2 By Equation 3, tiny 23. By Equation 1, tim = 7°(§), where f(x) = sinz. +1) 7 24. By Equation 3, tim, (0), where f (x) = 3" 2-3] LC+N=~F2) _ jin, [e+ -62 4h) i =e i (44 4h W192 6h~3)— (19) _ P= 28 2. 02) = /'Q) = fim = fim 4-2) = -2 m/s mt BoQasa= jim FOL LOE) _ jg BALAP- 24H +1]~ POP —24 1) I = Sh h _ on + $ 12H? + 24h + N= 2—H4 IIS _ 4 2H 4 1H? +234 =i # = z = fim (242 =23 m/s = Jim (20 4 12h +23) = 23 my SECTIONS1 D=RNAIVES O97 2. (@) f(x) isthe units are dollars per ounce. of change of the production cost with respect to the number of ounces of gold produced. Its (b) After 800 ounces of gold have been produced, the rate at which the production cost is increasing is $17/ounce. So the cost of producing the 800th (or 801s1) ounce is about $17. (©) Inthe short term, the values of /” (x) will decrease because more efficient use is made of start-up costs as x increases. But eventually /" (x) might increase duc to large-scale operations. 8. (a) f" (5) the rate of growth of the bacteria population when = S hours. Its units are bacteria per hour. (b) With untimited space and nutrients, /* should inerease as 1 increases: so f”(5) < J" (0). Ifthe supply of ‘nutrients is limited, the growth rate slows down ut some point in time, and the opposite may be true. 24. (a) f" (0) is the rate at which the fuel consumption is changing with respect tothe speed. Its units are (cal/hy / (ih) (b) The fuel consumption is decreasing by 0.05 (gal/h) / (mi/n) as the car's speed reaches 20 mi/h. So if you increase your speed to 21 mi/h, you could expect to decrease your fuel consumption by about 0.05 (gal/h) / (if). 30, (a) /”(8) isthe rate of change of the quantity of coffee sold with respect tothe price per pound when the price is $8 per pound. The units for /” (8) are pounds/ (dollars/pound) (b) J” (8) is negative sinee the quantity of coffee sold will decrease as the price charged for it increases. People are generally less willing to buy a product when its price inereases. 431. C’ (1980) is the rate of change of U.S. cash per capita in circulation with respect to time. To estimate the value of CC’ (1980), we will average the difference quotients obtained using the years 1970 and 1990, _ C1970) ~C (1980) _ 265-7 _ 5g, €.(1990) ~ C (1980) _ 1063 $71 _ fea oro 1980 10 OMB Gy gg Then C' (1980) = tim, CO C0980) AB _ 399 dottars per year 1aiBso = 1980 2 32. For 1910: We will average the difference quotients obtained using the years 1900 and 1920. “eg £1900) E910) 483—SL1 tet d= T900 = 1910 x19 = 028 and (1920) = £1910) _ 552-511 2A SU gat then 1920 — 1910 io OAT E@-E(90) 448 FOO) NB a t910 at about 0.345 years/year in 1910. For 1950: Using data for 1940 and 1960 in a similar fashion, we obtain £” (1950) ~ [0.31 + 0.10] /2 life expectancy at birth was inereasing at about 0.205 years/year in 1950. 1.345. This means that life expectancy at birth was increasing 0.205. So 38, Since (5) = wsin(1/x) when x # Oand f (0) = 0, we have £0) = fim FORA LO) fg BUI =O hig inci). This limit doesnot exis since sin (L/h) takes the values —1 and | on any interval containing 0. (Compare with Example 4 in Section 22.) 9 5) CHAPTERS OFAIVATIVES ‘3. Since f (x) = x? sin (1/x) when x £ Oand f (0) = 0, we have hy = P sin (1/) — fosn £0) jim” in (1/4) = 0 £0) = jim, jim fsin (1/h). Since 1 < we have \ 1 = al < fisin © < [hl Because lim (~ Yl) = Oand fim J] = 0, we know that tim (4+sin 2) = 0 by the Squeeze Uhl = sin « [hl cause fim (= Ya) = Oa fi, that ( i) Oby the Sq so ‘Theorem. Thus, /” (0) = 0. 1. Note; Your answers may vary depending on your estimates. From the ‘graph of fit appears that @ (ys 2 £2) 408 OL B~-1 (d) f' (4) = -0.5 The Derivative as a Function 2. From the graph of f. it appears that (9) /O)*18 £08 wOfa@xo £'G)*=1 (©) Aye 25 Os G)*0 3. It appears that / is an odd function, so f” will be an even function — " that is, f”(-a) = f(a). (PED =1S Wreyet Orcenxo ( s'@~-4 rao OF! @) /@)=15 4 J. since from left righ, the slopes of the tangents to graph (a) start out negative, become 0, then positive, then 0, then negative again, The actual function values in graph If follow he sare patter (by = 1V, sinee from left to right, the slopes of the tangents to graph (b) start out at fixed positive quantity, then suddenly become negative, then positive again, The discontinuities in graph IV indicate sudden changes in the slopes of the tangents, wo ince the slopes of the tangents to graph (c) are negative for x 0, as are the function values of graph | (6)! =I, since from left to Fight, the slopes of the tangents to graph (d) are positive, then 0. then negative, then 0, then positive, then 0, then negative again, and the function values in graph III fotlow the same pattern SECTION32 THEDERVATIVEAS AFUNCTION C9 n. 14, See Figure 1 in Section 3.5. 00 © CHAPTERS nEAIVATVES 18, (a) By zooming in, we estimate that #"(0) = 0. 4” (4) = 1, 7.0) =2, 25, and f° Q) = 4 () By symmetry. f" (=x) = = 7" (8). $0.9" (—$ J (1) = Rand f'(-2) = 4 (©) Ieappears that” (x) is twiee the value of x, so we guess that, J” (x) = 2x, as LEFM=FO)_ 4, @ HW? i Mo hQx thy a (4) /"() = fim tim ab) 16. (By zooming in, we estimate that" (0) =0, 4” ($) 0.95, © , S” (1) = 3, f" (2) = 12, and f” (3) 27. (6) By symmetry, (=x) = J"). So J" (—H) 0.75, /"(-) 3, S' (2) = 12, and 7" (—3) = 27. (d) Since " (0) = 0, it appears that /” may have the form f* (x) = ax? ae + Using f” (1) = 3, we have a = 3, 50 f" (x) = 3x7. 8) jig EAP oh (84322 434k? +h) 8 7 eS) Ci HO SHI) 2 = fm, : = fim (x2 4 3x41?) = 1 egy = tim LEEMALG) — ji OHIDF BI Ge43) Sh PO) Re HS Domain of f = domain of /” m LEM = SO) _ 4, S— AGW) +3 Ue + AP] ~ [S— Ae + 397] * i = sh A fy SAN MBs? 4 wt 42] = [5— Ae $327] a) 7 =th+ Oxh+ 3h? wy =fi = jim i Tim (44 6x + 3h) = 4-4 6 Domain of f = domain of J” = R. (<3 x? +2) =se othe +h? 1. 7") = fim FE fo) Jin, & +hy thrs2e +h) gg BETA BA? + HP = Qh = 1? + 2h > > = lim 7 = Jim (x? 38h 4? = 25h +2) =3x? = 2042 Domain of f = domain of f= SECTION 32 THEDERVATIVEAS AFUNCTION 101 _ 0 waneverd = (xt ve) | ER- VE Verh VE sb it h Vath Ve, “aa) Tae tae Domain of f= (0,28), domain of f° = (0,90) AFER EH — VOI [ VTETEFH + VIF Vist A+ Vis a els +M—£(9) o (1+ 2x + 2h) ~ + Ye ee es oo 2. £6) =f Domain of ¢ =[—4. 0), domain ote’ = (~$ , xthbt etl 2 10) = pm FETAL = yy EET = tim SHED DODO FAD _ =th iin, he th—-)@ ~bheth-De—) iy 2 be Fh= Ne 1 Domain of f = domain of f/ = (x Lx # 1) 3A 2+x)- (4 . 10h ray hQex HQ Tbh @txtHO+N = im "0 =o m>@txtHG+H QexP Domain of G = domain of G’ = {x [x # 2} L ox = glx) (x GHP - hy 2a) = fin EE gC on ed Sy et ra i mE hee eee HD eb 22x ‘Domain of ¢ = domain of g’ |x #0}. m & tay (4 + Or deh? $18) = 4% Domain of = domain of /’ = R. 102 © CHAPTERS DERIVATIVES B. (a) y= ome (b) Note thatthe third graph in part (a) has small negative values for is slope, f"; but asx > 6", f"—» oo. (OF) = lim LEAH = SO) = tim YE FH - Vo JF) + Vo=x h nao h VO—@ FH) + Vo—x [6~@ +M1~6=») t = tim ————! W350 Je=x=h+ Ve=3 Wo=% Domain of f= (00, 6], domain of ” = (—20, 6). ea aN oe er oy @ ” [Gayle +h z 2. (0) $2) = Jim FEAM= LO) a na? a tim | FD | © fy [4 2PM = is i ~H|' aerm =1+2e2 (b) Notice that when f has steep tangent lines, /* (x) is very large. When f is flatter, (x) is smaller. SECTION32 THEDSRIVATVEASAFUNCTION C103 6 6 ; Teeae TH? 6462 6- 60h 2.0 = Leet iy THR = 6-604 HF £0 = jim, 7h i Wb a+ G+ P|) , oh 24 of Cen?) sey This corresponds ~MbAI pati] (Iee) (b) Notice that has a horizontal tangent when ¢ = to /' (0) =0. /"is positive when fis inereasing and negative when Lf is decreasing, 5°(1980 for small values of h For 1978: 5° (1978) = =3.05 S(i980) _ 275 1980 (1982) ~ $(1980) h=2 > s' (1990) = SU%2)= S980) (1980) 1982 — 1980 2 So we estimate tha $"(1980) =} (3.05 ~ 0.20) = —1.62s. + [19% 1980 198219881986 T0819 1919941996 SQ | 305-1625 0.675 0575 0.15 0.10 0205 0075 125 140 104 Cl CHAPTERS DERIVATIVES (@) We could get more accurate values for (1) by obtaining data for the odd-numbered years, © 31. f is not differentiable at x = —L or at because there is a discontinuity there: and at x = 11 because the graph has vertical tangents at those points; atx because the graph has a comer there. 32, (@) g is discontinuous at x = ~2 (a removable discontinuity), atx = 0 (¢ is not defined thore), and at x jump discontinuity). Sta (b) g is not differentiable at the above points (by Theorem 4), and also at x = —1 (corner), atx = 2 (vertical tangent), and at x = 4 (vertical tangent), 33. As we zoom in toward (—1, 0), the curve appears more and more like ‘a straight line, so / is differentiable at x = —1. But no matter how ‘much we zoom in toward the origin, the curve docsn’t straighten out we ean’t eliminate the sharp point (a cusp). Sof isnot differentiable at x = 0. 34. As we zoom in toward (0, 1), the curve appears more and more like a ble at.x = 0. But no matter how much, "straighten out is not (ine, 90 fis differ ‘we zoom in toward (1, 0) or (=1, 0), the curve doe — we can’t eliminate the sharp point (a cusp). So Uifferentiable at x = Lim B@) St) 10, GR aIy GT a aa EY —— (0+) (0) £°(0)= jim FOF = LO) + This limit does not exist. and therefore 1” (0) does not exist ©) im ve = = cond f is continuous atx = 0 (e00t Function), so. fhas a vertical tangent at x=0. 2b 2 SECTION 2 THEDEAUATWEASAFUNCTION 105 eG)—s 0) _ lim 1, which docs not exist Oe O= by insite g(s)-2@) ag (12 — a3) (x12 4a) (©) 8 (a) = Jin, gS, ag = 1S ET al) (Dg VBA 4 aI) i xi pal a3 2 = I FETE BGI aD ~ 3998 = Fa (6) g (x) = x27 is continuous at x = 0 and (@) oa 2 lina GOL = fing sara = 0°. This shows that x haa vertical tangent line at x = 0, oa Y oa n negafOrt Be<8 PLOW =T 6 ite L@)- 10 k-61-0_ | x-6 _ M6 NR He TAM mo eT , But lim L0- LO _ im. Kool-o tim 2=% BRS TA ee TAI q ~t y- “1 ite<6 - 80 (6) = tim FO=LO goes not eist, However #65) = 6 x6 1 ifx>6 38. f (x) = [x] is not continuous at any integer n, so f is not differentiable at n by Theorem 4. Ifa isnot an integer, then / is constant on an open interval containing a, so_f’ (a) = 0. Thus, F/G) = 0, x notan integer. ifx 20 39. (a) f&) = x |x! 2 it, <0 (b) Since f (x) = x? for x > 0, we have f” (x) = 2x for x > 0. [See ite < Exercise 3.2.15(d).] Similarly, since f (x) = —x? forx <0, we , have f’ (x) = —2x forx <0. Atx = 0, we have 1 (0) = tim LEZ LD — hg E11 tine = 5 LO = Bao a = sah =o = So / is differentiable at 0, Thus, f is differentiable for all x. 108 CHAPTERS DERIVATIVES 40. (a) £20) = im 0" 2 (4) = fim, LORD LO =) = tim t= LO hot hoot ACh) oe Th ' 0 ife <0 OsM=75 WO-x) ifr>4 oo) x Mcx<4 ‘These expressions show that / is continuous on the intervals (co, 0), (0,4), 4, 5) and (5, 26). Sinee 15,1 0)= Jig, G—)=5 #0 om £00. Ise does not exist, so / is discontinuous (and therefore not differentiable) at 0. lim, ans and fis continuous at 4. Since f (5) is not defined, f is discontinuous at 5. (8) From (a), / is not differentiable at 4 since /* (4) x J (4), and from (c), f isnot differentiable at 0 or 5 1,0 tim £ ACA we have tim f(x) = fim (S—s)= 1 and tim, ft ra 4M, (a) IP f iseven, then a Lex +h) fx) L0 F(x) Fa fi = hi f(r =H) = £6) oy ae tim LEFANAL = jim, fet ax =—n} =~ Jim, a =-/'@) Therefore, /” is odd. (6) If fis odd, then x) a tim LEAMA LY _ 4 f= 4/6) LC9= jim hi =i i = tim LEAM=LOD iy = tim LOE A= S00) =f C(t roid = gt ax =f) ‘Therefore, ("is even 42. (a) © avi 1 (b) The initial temperature of the water is close to room temperature because of the water that was in the pipes. ‘When the water from the hot water tank starts coming out, d7/at is large and positive as 7 increases to the temperature of the water in the tank, In the next phase, dT/dt = as the water comes out al @ constant, high temperature. After some time, d7’/dt becomes small and negative as the contents ofthe hot water tank are exhausted, Finally, when the hot water has run out, d7'/dr is once again 0 as the water maintains its (cold) temperature. SECTIONS DIMERENTIATON FORMULAS 107 From the diagram, we see that the slope of the tangent is equal to tang, and also that 0 < < §. We know (see Exercise 15) that the derivative of f(x) = is /* (x) = 2x. So the slope of the tangent to the curve at the point (1, 1) is2. So. isthe angle between 0 and tan“! 2 = 63° § whose tangent is 2, that is, =333 Differentiation Formulas Ls@=sr-1 > f(y=S-0=5 2. Fix) = axl? = F/G) = =A (10x01) = 40? Bsa PoaP4i = =381 3 RP Ns 2a32-o +2 7. F (x) = (16x) = 4096x9 =F” (x) = 4096 (3x2) = 12,2882? 8B Hs) =(/2) =s9/B%=Bs> = HG) BYW=6% > YW 6 (-9) 718 = Sa RW =5Y = RO= HOw“ t mans 2e dx? is a constant 3 ro = Vidy? = RG) =-1V Tor 2g yn? / — 2 13. g(x) Pte? glade EQ =w-S w/w =v Ros ry PENG) EO ENA Multiplying fist: » = (x? + 1) (x3 +1) ete 3e2 4 Ont 428 3x? + 2e, SxS txt eat] y's Sef 43x? + 2x (equivalent) 110 © CHAPTERS OERWATIS B. f(s) =32 53343 = PG) =3: 15x45 5 (Ah -6 =9 Note that J” (x) = 0 when has a horizontal positive when / is increasing, and j” tangent, is negative when is deereasing. Ba 138 19) t M6. (a) 14 a7. (a) 30 0 Bx? 45x! — 15x? Wf @eaxtisextx! = P@=al= ‘Note that (x) =O when / has a horizontal tangent, {” is positive when J is increasing, and is negative when Ff is decreasing. The endpoints of f in this graph are about (1.9, 1.2927) and (2.1, 1.3455). An estimate of /* (2) is LMs5= 1.2997 _ 0098 2088 — 9.264, STS ) Siar > fy FQ) =2/ (8-2) = 0.263002. 2/(5x9) The endpoints of f in this graph are about (2.9, 1.19706) and G.1, 1.33932). Am estimate of /” G) is 1.33932 — 1.19706 _ 0.14226 3isas oe OTS . eat OSoyax—VF = PE)=l—prPat- se S@)=1- sy wom & From the graph in part (a), it appears that /” is zero at xy © 1.25, x9 0.5, and x5 ~ 3. The slopes are negative (50 is negative) on (—00, x1) and (x2,.x3). The slopes are positive (so Fis positive) on (s1, x2) and (3,90). SECTION33 DIFERENTIATION FORMULAS S109 ws , 2-30) (45) “7-9 7% Q-39 Maa Ya Wh =P 420? ayer eis) () . (+ery arth 4) (erta)la)—laxr tb)le) _ cer tad—acr = be _ ad = be ere eres etd? Oa ertay 38.) a. 5 = Ji(e— Ji-41) = Another Method: Use the Product Rule AP (2) = apn" bay a" bee bone? $aix tag => PY) = nays V4 (n= Vag ant? poe + Day bay 42 £G) ney = LMI en £0 Notice that the slopes ofall tangents to / are negative and /”(x) <0 always, 110 CHAPTERS DEAIVATIVES 4 fy a3 S43 Si (x) = 3+ 18x — 5.32? = 45! = 1502 Note that /” (x) = 0 when f has a hi tangent, /”is positive when / is increasing, and J” isnegative when f is decreasing 45 (a) 8 Note that /” (x) =O when f has a horizontal tangent, /" is positive when / is increasing, and /” is negative when f is deereasing. “The endpoints of / in this graph are about (1.9, 1.2927) and (2.1, 1.3455). An estimate of f” (2) is Lise Lat _ 9008 — 9.266 We) SG) = > P= BV =2/(5x™9) SQ) =2/(5-29) = 0.263902. ‘The endpoints of / inthis graph are about (2.9, 1.19706) and G.1, 1.33932). An estimate of /” @) is 1.33932 = 1.19706 _ 0.14226 ‘— ont ons. 1 (S@)=x-VE = f'x)al— pe a £@= Lew gig ¥ 0713. From the graph in part (a), it appears that /” is zero at xy © 1.25.2 0.5, and x5 * 3. The slopes are negative (so ” is negative) on (~00, x1) and (x2,x3). The slopes are positive (50 J” 8 positive) on (x1, x2) and (x3, 00) SECTION DIFFERENTIATION FORMULAS 114 38-6 47430 4 40 499 1 47 Se = 40 48. (a) (b) From the graph in part (a), it appears that gis zero at x = 0. The slopes are negative (s0 ¢/ is negative) on (—00, 0). The slopes are positive (so 2’ is positive) on (0, 20) Oe@= vei = 1 ge) == 7 . +r (241)? “4 [NT =i © AD@-eN 2 oe = RE Ai Daal = fan an line * rip Tre AGS Dox! = fad an equation ofthe tangent (= Dory beef 1 at o( ~ VF) , a @+)-@x) lax 50. => = OS —F OS ee * * wD WEetD DRErHe MOOD ¥ = —7G5 = —0.03, and an equation of the tangent line is y ~ 0.4 = —0.03 (x ~ 4), oF y = ~0.03s + 0.52, BL y=SQ)SrtV¥ 3 P)=14 4x77, Sothe slope of the tanger ne at (1,2) is," (1) = 1+ $ (1) = 3 and its equation is yrt=$6-Dorvabah 412 CHAPTERS DERUATIVES BR@r=aSM=— a 2 O= tangerine atte pint (1 equation isy— $= }(e4 ory = $241 B@y=/O= Ta = (b) 075 weejiere9 [ye “y= = So the slope of the (~~ © (42) (4x) a / tangent line atthe point (0.3) is" 3) = 7 andits equation is J) = 0.3 = —0.08 (x —3) or y = -0.08" 40.54. os BB. (a) (fg) (5) = f S).8' (5) +85) S/S) =) 2) + (-3) 6) = 2-18 = 16 ‘ "6)— f 6) (5) w (2) o=22Lo-s02o gp or £Y') - £08 O-8 oo VOF 86. (a) (f +2) G)= SG) + 2B) =-6+5=—1 (b) (7a) 3) = £B) x (3) + 8B) £B) = 4) (5) + (2) (—6) = 20-12 =8 LY gy = LOLM=LO2B) _ DWEH- HE) _ =” @ (LY @ = EOL M=LO2@ _ OCH“ WE) _ =B __g ‘ (5) ° wor 2 4 LY gy Y@=2B1/'O)-FO[LB=-#B) _ G-DCO-4(-6-9) © (A) ° 1@)-e@P (4-27 2A =8 51. f(x) = Fg) 9 f') = VEE) +80) pr 7,90 SF = Ag +94) sh =2-748 = Las (0) v6) = Fa) /g 2), 500! (6) = LOLOL TNS OH @y=N SOY > =H) + SEA (x) 2 f(x) =f) Ox y= - P= F190) * (y © x2 = £4) Ax) ~ x7 f(x) ** TO Vor SECTION 33 DIFFERENTIATION FORMULAS =O 113 L4af@) Ore Vf) +S) -11 + Olag (va? BAP 4? f(x) — Sat? F Ox) 2x!2 _ xf (x) #27 F x yi = Bee Fo) = so slope (2) =—4. The normal line has slope — 2, so the tangent line at (2, ~3) has 1 4 and equation y43—}O-2or &yp=fw)ar—28 3 fe) =1-2%. So the normal lin isthe negative reciprocal of that of the tangent line, that is, =1/($1)= |. So the equation ofthe normal line at (1,0) is y ~O= I (x = 1) ~ 1. Substituting this into the equation ofthe parabola, we obtain 1, and the slope of ey O for all x, som > 5 for all x 2s 4-1. Ifthe point at which a tangent mets the parabola is (2, ++), then the slope of the tangent is 2 + 1. But since it passes through (2,—3), the stope must Therefore, datt= cee Solving this equation fora we get a? +a43=2a?—3a-2 © ada Yat =0 of =I. Ifa = =1, the point is (1,0) and the slope is~1,s0 the equation is y (- + Dory= |. [fa = 5, the point is (5, 30) and the slope is 11, so the equation: ky 302 11(¢ = Sony = Mx 25 We will sometimes use the form fs + J rather than the form Ji’ + gy forthe Product Fle fa) AY = (fay b+ (fab = (f'e + fia) ht fayh! = f'gh + fat + Sah! 68. (a) (fein = 70. (a) Putting / 2 = hin Exercise 69, we have LILA ILS + IL =f SI OFS &). Atroor =u (by y= (0 3x4 17 482) ey 3 (et 4 ae 4 1 482)" (4 4 982 +17) SECTION DIFFERENTIATONFORMULAS OB Wy =(()=a8 +b? bextd > /'Q)= 3a 42 4+e (-D=6 = Ba $46 = 2e4+d =6(1). £2) =0 => 8a $44 2e-+d=0 2). Since there are horizontal tangents at (-2,6) and (2,0), /” (42) =0, f"(-2)=0 = 12a—4b +e =0@)and /7Q)=0 = Pa ttt c= 0(4) Subiracting equation (3) from (4) gives $= 0 = 6 =0. Adding (1) and (2) gives 85 + 2d = 6,s0d = 3 since b =. From (8) we have.© = ~12a, 0 (2) becomes Rar + 4 (0) + 2(—12a) +3 = 0 12a = 12 (jg) =] and the desired cubic funtion is =3 6a > a=. Nowe = ya gears 72 (4) + (0) = 10,000 suseribers and (0) = 1.2 phone lines pr subscriber « (0) = 1000 subscribers month and nf) 0.01 phone line per subscriber/month, (b) The total number of lines is given by L(t) = s (1) (0). To find L/(0). we first find L' (0 using the Product Rule. £’() = s(n) +nOSO 1!) = (0)! O) +m O)s” (0) = 100,000 (0.01) + 1.2 (1000) = 2200 phone lines /month, 73, Let P (0) be the population and let 4 (2) be the average 1 = 0 corresponds to July 1993. Then the total personal income is given by T () = P (1) 4 (1). We wish to find TO). TO = POA WA] P’ (0). The term P(A repees income due to the existing population’s inereasing income, The term A (1) P(e) represents the of the rate of change of total income due to the inereasing population. PO) AO) +A) P*O) © G,354,000) (1900) + (21,107) (45,000) Personal income was rising ata rate of about $7.322 billion per year, il income at time /, where ¢ is measured in years and the portion of the rate of change of total ,322.415,000, So the total 7A. (a) £20) = 10,000 means that when the price of the fabric is $20/yard, 10,000 yards will be sold. F” (20) = ~350 means that as the price of the fabric increases past $20/yard, the amount of fabrie whieh will be sold is deereasing at arate of 350 yards per (dollar per yard, () RD) = PFD) SL(p) 1 > REQ) = 20/" (20) + f (20) 1 = 20(—350) + 10.000 = 3000. This cans that as the price ofthe fabri increases past $20/yard, the total revenuc is inercasing at $3000/ (S/yand). Note that the Produet Rule indicates that we will lose $7000/ ($ yard) due to selling less fabric, but that that toss is more than made up for by the additional revenue due to the inerease in price TB. f(s) =2 Hil < Land f(x) = x2 = 2x 4 2ife > 1. Now we compute the rights and left-hand derivatives defined in Exeeise 3.2.40: Lem 7 =U h=1 A £0) = tim FOF MA LO fgg 2 HEAT Vand a i wae £U+h h A) = = im, = tim, =o. FO = I, io pee Thus, £0) does not exist since f° (1) & J. (0. , 4 0 f isnot differentiable at 1, But "(x)= =1 h Vy x) =2e-2ife> 1 | for x < Vand /” 118 © CHAPTERS DERNVATIUS ba2y ity c= Bam={ ee f-terst + ifsd jim SEEEMT ECD hig ay a ne fim SEEM 8D yg LEMP T got A not h ~ so g is differentiable at —1 and g’ (—1) tim EC4+M= 2 and at h gQ+h) ny = lim +1) = slim |. $0 g! (1) docs not exist. lim, ‘Thus, g is differentiable exeept when x = 1, and -2 ifx<-t ws] 2 17 (@) Notethat x? 9 < Ofer? <9 ee be] <3 @@ —33 Toshow that /“@) des not exit we neste tig ! B= £0) gy capping the eam ihchand derivatives defined in Exercise 3.240 ap bees O49) (y= tin LEFDA LO) gg EOF D=0 gy 6-44) = and ig an r lig -0 yh 1.8) = tin, LEED ALO) _ jig LOEWE O gg HHH im 6-4 A) = 6 ho A hoot h poor oh hor Since the lt and ight limits ae diferent ® 64h) i 1/3) does not exist. Similarly, /* (—3) does not Therefore, f is not differentiable at 3 or at SECTIONS DIFERENTIANONFORVILAS 117 TB Ife > Vy then h(x) = |x = M4 42] =a— 14x $2 =2K41 If-2 1 cexist, observe that fs) — hl) ti tim at xt tim, lim, x Similarly, i! (—2) does not exist, 79. y= f(x) = ax ‘The slope of the given line is seen to be ~2, so we musthave 4a =-2 3 a =f" (x) = 2ax. So the slope of the tangent to the parabola at x = 2 is m 1a (2) = 4a, So the point in question 2. Now we simply require that the given line, whose equation is 2x + y = b, pass hhas y-coordinate through the point (2,—2): 2(2)-+(-2)=b © b=2. Sowe must havea =—$ and b= 2. 80. / is clearly differentiable for x < 2 and for.x > 2. Forx <2, /’(x) = 2x,s0 f. Q)=4, Forx > 2, 80 (2) = m. For f tobe differentiable at x = 2, we need 4 =f" (2) = f,Q)=m. So 4x +4, We must also have continuity at x = 2, s04= 72) lim, (4x +4) lim fe aot 4 Hence, b = Vie 8 Fethg =F = fete =f S. msoquton is 20 Lot P= (a,£), Tsipsorthetnget in at = a8) + =, so its -ayory =~ = 2a, sothe 2x intercept is —. Setting y = 0 gives rept is 2a The midpoint ate fn eget joining (0, 2) art Qa.0) 6 (0, ) = P (b) We know the x~ and y-intercepts of the tangent fine from part (a), so the area of the triangle bounded by’ the axes and the tangent is $y = } (2a) e/a) = 2c, a constant. 18 CHAPTERS DERIVATIVES 83. Solution 1: Let f(x) = x! Then, by the definition of derivative, L)= £0) x= T ee £03) = Lim But this is just the limit we want to find, and we know (from the Power Rule) that /” (x) = 1000, so /' (1) = 1000 (1) = 1000, Solution 2: Note that (x! — 1) = 1) (9? TP ba FD). SO hn sg SED ooo tN) bey tim lim (°° 9 4 bee) Lee eek Led 1000 ones = 1000, as above. In order for the two tangents to interseet on the y-axis, the points of tangency rust be at equal distances from the y-axis, since the parabola y = x? is symmetric about the y-axis. Say the points of tangency are (a,®) and (2,02), for some @ > 0. Then since the derivative of y =x? is dy/dx = 2x, the left-hand tangent has slope ~2a and equation y — —2a (x +a), or y =—2ax — a, and similarly the right-hand tangent line has equation y= ar ~ 2 So the wo fines intersect at (0 a2), Now if the lines are perpendicular, then the product oftheir ) slopes is ~1, so (~2a) 2a) o> a=} o a=}. Sothelines intersect at (0, 241 Rates of Change in the Natural and Social Sciences L@s=s0) lor+12 > v=) =Cr-10)h/s (b) 0G) =2)= = -4 8s (©) The particle is at rest when 9 ()=0 € 2-10=0 @ 15s. (a) The particle is moving in the positive direction when » (1) > 0 €2 2-10>0 o 2%>10 es (¢) Since the particle is moving in the positive 0 direction and in the negative direction, we need to calculate the distance traveled in the intervals (0, 5] and [5, 8] separately. 1£ 6) — fW) = [=13 = 12] = 25 fland 18) = /(9)] =|-4 = (13) =9 A The total distance traveled during the first 8 sis 2549=34R
  • También podría gustarte